Last visit was: 06 May 2024, 21:07 It is currently 06 May 2024, 21:07

Close
GMAT Club Daily Prep
Thank you for using the timer - this advanced tool can estimate your performance and suggest more practice questions. We have subscribed you to Daily Prep Questions via email.

Customized
for You

we will pick new questions that match your level based on your Timer History

Track
Your Progress

every week, we’ll send you an estimated GMAT score based on your performance

Practice
Pays

we will pick new questions that match your level based on your Timer History
Not interested in getting valuable practice questions and articles delivered to your email? No problem, unsubscribe here.
Close
Request Expert Reply
Confirm Cancel
Magoosh GMAT Instructor
Joined: 28 Dec 2011
Posts: 4452
Own Kudos [?]: 28605 [0]
Given Kudos: 130
Magoosh GMAT Instructor
Joined: 28 Dec 2011
Posts: 4452
Own Kudos [?]: 28605 [0]
Given Kudos: 130
Magoosh GMAT Instructor
Joined: 28 Dec 2011
Posts: 4452
Own Kudos [?]: 28605 [0]
Given Kudos: 130
Magoosh GMAT Instructor
Joined: 28 Dec 2011
Posts: 4452
Own Kudos [?]: 28605 [0]
Given Kudos: 130
GMAT Sentence Correction with 20 Practice Questions and Answers [#permalink]
Expert Reply
FROM Magoosh Blog: GMAT Sentence Correction with 20 Practice Questions and Answers


Every time you put something into writing in a professional setting, it should represent you at your very best—and an important part of that is correct grammar. That’s why business schools care about it, which is why the GMAT tests it. The primary vehicle for testing grammar on the GMAT is the Sentence Correction questions. Keep reading to see what is tested on GMAT Sentence Correction, practice questions for SC, and tips for success. ?

What is GMAT Sentence Correction?[/*]
[*]How can I do well on SC?[/*]
[*]How to Master Sentence Correction on GMAT
[*]GMAT Sentence Correction Practice Questions with Explanations[/list]


GMAT Sentence Correction Strategies.

While you’re thinking about strategy, here are some tips on how to ace GMAT sentence correction!



GMAT Sentence Correction Practice Questions is an excellent resource. It compiles links to other blog posts, listed by the rule that they have to do with. So, if you wanted to learn about gerunds and gerund phrases, or when to use like vs. as, you can go to a post that focuses on that rule with examples.

In many ways, SC improvement takes time. You have to learn and re-learn grammar and style rules. This takes time, so be patient with yourself. Use the lesson videos, and make sure to learn from your practice. Also, your process of elimination skills need sharpening and training. The GMAT is a rapid and tricky test, and the more organized and purposeful prep that you do, the more likely it is that you’ll see an increase in your practice scores.



GMAT prep and are a good sample of the different sorts of questions you’re likely to see on your exam. Plenty of different grammar and style rule violations, problems with the structure and logical flow, and some examples of issues with idioms and phrasing.

  • Getting adequate sleep, a full eight hours every night, the depth of which will be enhanced by a regular regimen of physical exercise, with significant consequences for not only one’s immediate short term health, but also for the immune system’s ability in fighting major illness over the long term.
    (A) with significant consequences for not only one’s immediate short term health, but also for the immune system’s ability in fighting

    (B) has significant consequences not only for one’s immediate short term health, but also for the immune system’s ability to fight

    (C) having significant consequences for not only one’s immediate short term health, and for the immune system’s ability to fight

    (D) has significant consequences not only for one’s immediate short term health, and also for the immune system’s ability in fighting

    (E) which has significant consequences not only for one’s immediate short term health, and the immune system can

    Show Answer and Explanation
    Click here for the video explanation from our GMAT product.

    First of all, the GMAT loves the construction “not only X but also Y.” (A) and (B) do this correctly, but (C), (D), and (E) all use the idiom incorrectly. (C) and (E) use “not only … and” and (D) says “not only … and also.”

    Another important split involves a parallel structure. The word “for” can either appear once before and outside the parallel structure— “for not only P but also Q” (no answer choice has this) — or it can be repeated in each term of the parallel structure —- “not only for P but also for Q” (choices (B) & (D) have this correct). It is a classic GMAT Sentence Correction mistake to have the common word once outside and once inside, as in “for not only P but also for Q” (choices (A) & (C) have this incorrect construction). Choice (E) violates the parallelism, so that’s even worse!

    The third split is that “ability” has to follow an infinitive, such as the “ability to do something.” Choices (B) and (C) do this correctly. A common mistake in the use of this word involves the word “in,” such as “ability in doing something.” (A) and (D) make this mistake. (E) avoids this issue by violating the parallel structure which as we saw, is its own problem.

    The fourth and final split involves the missing verb mistake. If you notice, choices (A) & (C) are not complete sentences — the subject “getting” does not have its own verb anywhere in these sentences. Choices (B) & (D) provide the main verb “has” for the main subject “getting.” Sentence (E), with its substitution of “that” for “which” has a particular bizarre structure — technically, in version (E), everything from “the depth of that” to the end would be a complete sentence, but then the whole first part is odd gerund phrase that sticks out awkwardly like a sore thumb and doesn’t fit with the rest. If we fixed the other mistakes in choice (E), we could rectify this problem by constructing two separate sentences following this layout: “Getting adequate sleep, a full eight hours every night, is important. The depth of this sleep will be ….” That would be a possibility in real-life editing. In the GMAT Sentence Correction, though, you have to stick to one sentence, so (E) is out.

    The answer is (B).
  •  

  • Internal combustion engines show an inherent tradeoff between power and fuel efficiency: if a car can race from zero to 60 mph faster, the less fuel-efficient its engine is over time.
    (A) if a car can race from zero to 60 mph faster, the less fuel-efficient

    (B) racing from zero to 60 mph faster, it is that much less fuel-efficient

    (C) the faster the car can race from zero to 60 mph, the less fuel-efficient

    (D) the faster the car can race from zero to 60 mph, it is that much less fuel-efficient

    (E) when the car can race from zero to 60 mph faster, the less fuel-efficient it is

    Show Answer and Explanation
    Click here for the video explanation from our GMAT product.

    This sentence focuses on a unique idiom. This idiom is used for comparisons that describe how two quantities change — both getting bigger together, or both getting smaller, or one bigger & one smaller, etc. The structure of this idiom is

    the [comparative adjective][clause #1], the [comparative adjective][clause #2]

    This is a full, independent clause on its own, and does not constitute a run-on sentence. Also, the words “the” are crucial to this idiom. It’s a very strange idiom, almost certainly one that will confuse non-native speakers. Fortunately, it always has the same structure, so you can learn how to use it.

    In this sentence, what are the two comparative adjectives? The first one is “faster”, a single word, and the second, slightly longer, is “less fuel-efficient.” The general format, therefore, should be

    the faster ……, the less fuel-efficient …..

    After each comparative, we should have a full [noun] + [verb] clause. Choice (C) is the only one that follows this pattern. All the others — the “if”, the “when”, the “that much less fuel-efficient” — all of these variants are incorrect. This idiom has a very precise structure, and only choice (C) reproduces this structure correctly in both halves of the sentence. Sometimes that’s all you need to find the answer!
  •  

  • Unfairly criticized during his time, Emperor Claudius expanded the Roman Empire to the east, constructed public works such as aqueducts, having passed laws for protecting the rights and freedoms of Jews throughout the Empire.
    (A) having passed laws for protecting

    (B) having passed laws to protect

    (C) having passed laws that protected

    (D) and passed laws by protecting

    (E) and passed laws that protected

    Show Answer and Explanation
    Click here for the video explanation from our GMAT product.

    The first split involves parallelism. We have two verbs in parallel in the non-underlined portion of the prompt, “expanded …. constructed …”, both simple past tense, so we need the third verb, in the underlined section, to be parallel to these. Therefore, we need the simple past tense “passed”, so (D) & (E) are correct. Choices (A) & (B) & (C) instead have the participle “having passed”, which is not parallel at all, so these three are wrong.

    Next, we have a three-way split in the word following “laws” — “for”/”to”/”that”. One correct approach is to use a “that clause” — “laws that protected.” Here, the verb “protected” describes what the laws do. Choices (C) & (E) have this correct structure.

    Choice (B) takes a different approach that is still correct. It uses an infinitive of purpose, which modifies the verb “passed laws to protect …” The infinitive “to protect” gives the reason why the laws were passed, so this is correct.

    Choice (A) tries to express purpose using the structure “for” + [participial phrase]. This is not an idiomatically correct way to express the purpose of an action, so this choice is incorrect. An infinitive of purpose would be the correct way to do this.

    Choice (D) changes the meaning by changing the logic. The purpose, the goal, of passing the laws was to protect the rights of Jews. The rights of Jews were not the cause of anything, which is what the “by” suggests. This choice is incorrect.

    Because of these two splits, the only possible answer is (E).
  •  

  • Since around 1970, with world population of 3.5 billion, they have doubled it to our current level of over 7 billion in 2012.
    (A) with world population of 3.5 billion, they have doubled it

    with a world population of 3.5 billion, doubling it

    (B) when the world’s population was 3.5 billion, doubling it

    (C) when the world’s population was 3.5 billion, it has doubled

    (D) when the world’s population were 3.5 billion, they have doubled it

    Show Answer and Explanation
    Click here for the video explanation from our GMAT product.

    One split is the “with” phrase vs. the “when” clause at the beginning. “When” is the correct word to use here because we’re describing the year 1970, and that’s time. Compare that to the prepositional phrase “with X” has to modify the thing that has X. Here, the phrase is “with a world population of 3.5 billion”, and this raises the fascinating question: what is the thing, or what could be the thing, that has “a world population of 3.5 billion”? It doesn’t make sense to refer to the year 1970 this way, and there’s nothing following the comma that is a likely target, so this preposition phrase structure is problematic. (A) & (B) are wrong.

    Now, let’s look at the split after the comma–we have “they have doubled …”, “doubling”, or “it has doubled.” The “they” is a mystery pronoun, a pronoun with no properly defined antecedent. The GMAT hates mystery pronouns. The choices with the mystery pronoun, (A) & (E), are wrong. The participle “doubling” is also wrong. The only circumstance where “doubling” could be right is if it were modifying some noun that was performing the action of doubling, and such a noun doesn’t exist in this sentence. Therefore, the choices with the participle “doubling”, (B) & (C), are wrong. This leaves only (D), which has an elegant structure that is 100% grammatically correct: “it [Earth’s population] has doubled.”

    (D) is the correct answer.
  •  

  • The more cautious commanders of the Army of the Potomac, such as George McClellan and George Meade, chose to withdraw after costly battles, but commander Ulysses Grant’s strategy was the keeping up of pressure on Lee’s Confederates even after losing a battle to them, and then he occupied positions between Lee’s army and Richmond to invite another open battle.

    (A) was the keeping up of pressure on Lee’s Confederates even after losing a battle to them, and then he occupied

    (B) was keeping up pressure on Lee’s Confederates even after losing a battle to them, and then to occupy

    (C) having kept up pressure on Lee’s Confederates even after losing a battle to them, he then occupied

    (D) was to keep up pressure on Lee’s Confederates even after losing a battle to them, and then to occupy

    (E) was that he keep up pressure on Lee’s Confederates even after losing a battle to them, and then occupying

    Show Answer and Explanation
    Click here for the video explanation from our GMAT product.

    Once again the split is based on parallelism. Grant’s successful strategy had two parts, and these two parts need to be in parallel. Grant kept up pressure, and Grant occupied a position. Those two need to be in parallel.

    (A) “…the keeping up of pressure … he occupied” = complex gerund (very awkward!) + full verb = a failure of parallelism. This is incorrect.

    (B) “keeping up pressure …. to occupy” = gerund + infinitive = a failure of parallelism. This is incorrect.

    (C) “having kept up pressure … he … occupied” = participle + full verb = a failure of parallelism. This is incorrect.

    (D) “to keep up pressure …. to occupy” = two infinitives in parallel = perfect!

    (E) “that he keep up pressure …. then occupying” = full verb + participle = a failure of parallelism. This is incorrect.

    That’s all the information we need to choose (D) as the correct answer.
  •  

  • The controversial restructuring plan for the county school district, if it is approved by the governor, would make there be fewer teachers in schools throughout the county.
    (A) if it is approved by the governor, would make there be fewer teachers

    (B) if the governor approves it, would result in the amount of teachers going down

    (C) if the governor approves it, would have the result of decreasing the number of teachers

    (D) if approved by the governor, would result in a reduction of the number of teachers

    (E) if approved by the governor, would decrease teachers

    Show Answer and Explanation
    Click here for the video explanation from our GMAT product.

    Let’s start with the phrase directly after the first comma, “if it is approved.” The phrase that comes after a comma often describes a noun in the phrase before the comma. Here, it’s describing the plan. If the plan is approved, there will be fewer teachers.

    Choice (A) doesn’t work because it uses the pronoun “it.” We don’t need a pronoun here. It just makes the sentence more confusing. Is “it” the plan? The school district? Ambiguity is bad on SC, so cross it off. (B) and (C) have the same problem. The pronoun “it” is unnecessary and ambiguous, so we’re left with (D) and (E).

    (E) “would decrease teachers”—very terse, but unclear in meaning; it seems to imply that the plan will make less of teachers as human beings, somehow “decreasing” their essential humanity! This is a sloppy and imprecise way to say what the sentence is trying to say. This choice is incorrect. Choice (D) is clear, direct, and elegant. We don’t need a verb between “if” and “approved” because an “it is” is implied.

    Choice (D) is mistake-free and is the best answer here.
  •  

  • Perennially the world’s leader in tea production, China’s fascination in tea has deep historical roots, as exemplified with the tea ceremony, which has analogs in Japan and Korea.
    (A) Perennially the world’s leader in tea production, China’s fascination in tea has deep historical roots, as exemplified with

    (B) China perennially has been the world’s leader in tea production, and its fascination with tea has deep historical roots, as exemplified by

    (C) Fascinating with deep historical roots, China’s tea production perennially leads the world, thereby exemplifying

    (D) Perennially the tea producing leader of the world, China has a deep historical fascination with tea, as an example of

    (E) China, perennially the world’s leader in tea production, and their fascination with tea has deep historical roots, as exemplified in

    Show Answer and Explanation
    Click here for the video explanation from our GMAT product.

    When in doubt, put the subject of the sentence first. That won’t always be true, but it’s a good rule of thumb. (B) and (E) both have China first, and it turns out that (B) is the answer. We’ll discuss that in just a moment, but first, let’s talk about exceptions.

    If we want to say something about China first, we have to do it correctly. (A) doesn’t work, so let’s see why.

    “Perennially the world’s leader in tea production, China’s fascination…”

    Whatever comes in the dependent clause at the beginning has to depend on the subject of the sentence. China is the world’s leader in tea production, so China must be the subject. In (A), China’s fascination is the subject. This doesn’t make any sense. How can fascination, as an emotion or state of mind, be a leader in tea production? (D) is a good example of how to start a sentence with an appositive phrase that leads up to the subject of the sentence. The problem with (D) is the phrase “as an example of.” The correct idiom is “as exemplified by.”

    We’ve crossed off (A) and (D), but what about (C)? The problem with (C) is that it changes the meaning of the sentence. (C) says that China’s tea production is fascinating to the world. It’s also strange to say “China’s tea production perennially leads the world.” If anything we’d say China perennially leads the world in tea production.

    Now we’re down to (B) and (E). Another rule of thumb is to pick the simpler of two sentences, and (B) is simpler. (E) has multiple clunky phrases not to mention its grammar mistakes. China is singular and yet it says “their fascination.” What’s more, if we’re saying China and [its] fascination, then we should say the plural “have” and not the singular “has.”

    (B) has two independent clauses that are combined correctly and free from error.
  •  

  • Walter Mondale had a long difficult struggle leading up to the 1984 Democratic Presidential Primary, though he consolidated his lead for the nomination, but he won only 13 electoral votes in the general election.
    (A) Walter Mondale had a long difficult struggle leading up to the 1984 Democratic Presidential Primary, though he consolidated his lead for the nomination, but he won only

    (B) Having had a long difficult struggle leading up to the 1984 Democratic Presidential Primary, Walter Mondale still consolidated the nomination lead, only winning

    (A) After a long difficult struggle leading up to the 1984 Democratic Presidential Primary, Walter Mondale consolidated his lead for the nomination, but he won only

    (D) A long difficult struggle leading up to the 1984 Democratic Presidential Primary, Walter Mondale, consolidating his lead for the nomination, but he won only

    (E) Walter Mondale had a long difficult struggle leading up to the 1984 Democratic Presidential Primary, consolidated his lead for the nomination, and only won

    Show Answer and Explanation
    Click here for the video explanation from our GMAT product.

    Watch out for contrast words! (A) has two which makes it nonsensical. Though he consolidated his lead but he won only 13 votes? When we set up a contrast we have to contrast it with something. We could have said, “Though he consolidated his lead, he still lost” or “His campaign was going well, but he only won 13 votes.” Both of these are potential contrasts because they say one thing is the case, then unexpectedly something opposite is the case. (A) doesn’t work because contrasting a contrast is ambiguous and awkward ?

    (B) doesn’t make sense in a different way. It makes it sound like the details in the sentence fit together. He struggled, consolidated the lead, only winning 13 votes. If things were going well, why did he only win 13 votes? Cross it off.

    (C) is the answer. After a difficult nomination, he consolidated his lead, but won only 13 votes. Those details make logical sense, and the grammar is fine too. Here is the structure:

    Preposition (“After”) + Noun with some Adjectives (“a long difficult struggle”) + Present Participle (“leading up to the 1984 Democratic Presidential Primary”).

    The present participle here acts as a noun modifier, modifying the noun “struggle.”

    If (D) sounds a little off to your ear, then well done! You’ve found the famous missing verb mistake. If you missed it, don’t worry! You can see that the sentence has a long absolute phrase, then the noun “Walter Mondale,” then a participial modifier, but this noun has no verb before the word “but” starts the second independent clause. The first independent clause doesn’t actually have a verb. “Leading” and “consolidating” are descriptions, not actions. One fix would be changing “consolidating” to “consolidated.”

    Finally (E), which has false parallelism. There are three clauses in a list separated by commas, but just because you put three verbs in a list doesn’t mean you have a grammatical sentence ? The trouble is that it doesn’t explain what the relationship between these clauses is. The correct answer (C) uses a prepositional phrase to explain that he was doing well, and after that he didn’t do so well. The logic is clear. (E) just puts the three details in a row but doesn’t express the logic.
  •  

  • Whereas both Europe and China use standard railroad gauge (1435 mm), Russia deliberately chose the wider “Russian gauge” (1520 mm) that gives greater side-to-side stability in railways cars and, more importantly, acts as a national defense, so that it would block a foreign army’s supply line and preventing these bordering powers from invading by train.
    (A) acts as a national defense, so that it would block

    (B) acts like a national defense, so as to block

    (C) acts as a national defense, blocking

    (D) acting as a national defense, blocking

    (E) acting like a national defense, would block

    Show Answer and Explanation
    Click here for the video explanation from our GMAT product.

    The first split has to do with the verbs “gives” and “acts.” Parallelism says that these two verbs have to be in the same form. “Gives” is simple present tense, so we need the same for “acts.” (D) and (E) don’t do this, so cross them off. (A), (B), and (C) remain, so we have to go deeper.

    You may recognize like vs as, but “act like” and “act as” are particular idioms, so need a little more explanation. You can read about that here: A Tricky GMAT Idiom. The idiom “act like” means the subject intentionally decides to imitate the attributes of something else; humans can “act like” something, and, under certain circumstances, more intelligent animals could “act like” something if we attribute some kind of intentionality to them. An inanimate object can never “act like” anything. The idiom “act as” means that, without intentionality, one object takes the role of another. Here, we are talking about railroad gauge, an inanimate object, which could never “act like” anything. We need the construction “acts as.” (A), (C), and (D) do this correctly, but we already crossed off (D), so we’re left with just (A) and (C).

    The final split has to do with parallelism again. The two verbs are “blocking” and “preventing,” which is the participle form. By the way, do you get mixed up between participles and gerunds? Participles act like adjectives and gerunds act like nouns. You can read about this and other tips here: Top Six GMAT Tips for Sentence Correction. “Preventing” is not underlined, so is ‘locked in.’ Whatever is underlined has to match it. Only (C) and (D) do this, so we know that (C) is the answer.
  •  

  • Even-toed ungulates, including pigs, cattle, goats, and sheep, and odd-toed ungulates, such as horses and donkeys, account for all the mammals domesticated for agricultural purposes.

    (A) including pigs, cattle, goats, and sheep, and odd-toed ungulates, such as horses and donkeys, account for

    (B) including pigs, cattle, goats, and sheep, and odd-toed ungulates, including horses and donkeys, accounting for

    (C) included among pigs, cattle, goats, and sheep, and odd-toed ungulates, such as horses and donkeys, account for

    (D) included among pigs, cattle, goats, and sheep, and odd-toed ungulates, like horses and donkeys, are accounted for by

    (E) like pigs, cattle, goats, and sheep, and odd-toed ungulates, like horses and donkeys, are accounted for by

    Show Answer and Explanation
    Click here for the video explanation from our GMAT product.

    Choice (A) is correct! No mistakes. It may help to have that in mind as we discuss what makes the other four choices wrong. (B) makes the missing verb mistake. It says “accounting for”, which means the noun isn’t doing any action. It’s just being described. That makes the sentence incomplete. Next, we can cross off (D) and (E) because they both use “like” to list examples. While English speakers do use this word to list examples in casual speech, it’s considered an error on the GMAT.

    The other two problems are more complex because they have to do with logic. The way that (A), (B), and (E) are phrased, even-toed ungulates is the group which contains pigs, cattle, goats, and sheep. Those four animals are examples of even-toed ungulates, the way that salmon and trout are both types of fish. (C) and (D) say that even-toed ungulates are included among pigs, cattle, goats, and sheep. The GMAT doesn’t require any scientific knowledge, but it does require you to infer relationships based on the words they use in the prompt.

    Finally, the idiom “to account for” means to explain, or show where something came from. For example, how can I account for the fact that I’m soaking wet? I walked here in the rain. Or, coffee accounted for more than half of all food purchases last year. The prompt says that these larger biological groups account for the relatively small group of mammals that are used for agricultural purposes. (D) and (E) reverse this. They say that the mammals in agriculture somehow account for the ungulates. That doesn’t make any sense, so cross these off.

    There you have it! A lot is going on in this problem. First, we have the usual grammar and style mistakes (the missing verb mistake and “like” for a list), but then we have two more subtle mistakes where an error shifts the meaning. It’s important to be on the look-out for all of these errors. They might violate a rule or just change the meaning. Both types of mistakes are a potential reason to eliminate an answer choice, so be on the look-out ?
  •  

  • The company is fortunate to have excellent relationships among its employees: they each have a relationship of respect for all the others.
    (A) they each have a relationship of respect for all the others

    (B) they have respect for one another

    (C) each one has respect for one another

    (D) they each have a relationship of respect for each other

    (E) they and the others respect each other

    Show Answer and Explanation
    Click here for the video explanation from our GMAT product.

    This problem includes indefinite pronouns, and those can be tricky. We have a post on this, and I highly recommend it: Indefinite Pronouns and Logic ? We need a phrase like “each other” or “one another.” That’s how you refer to both people in a two-person interaction without specifically referring to them. For example, start with a sentence with only specific nouns:

    The girl threw a ball to her friend.

    vs.

    They threw the ball with each other.

    The same basic idea is happening in our sentence. The employees have respect for each other. (A) does not use the indefinite pronouns correctly. It’s super wordy and awkward, so cross it off.

    The next split has to do with the structures “each other” and “one another.” They refer to more than one person, so we need a plural subject. (C) and (D) both make this mistake. (C) says “each one” and “one” is singular. Cross it off. (D) is more subtle, but by saying “they each have a relationship of respect” we’re talking about them individually. This is wrong, so cross off (D) as well.

    Now we’re down to (B) and (E), so what is the difference? (E) refers to “they and the other.” “They” refers to the employees, and there isn’t anyone else, so (E) is referring to nothing. Cross it off, and (B) is the answer.
  •  

  • Nine months after the county banned jet skis and other water boats from the tranquil waters of Puget Sound, a judge overturned the ban on the grounds of violating state laws for allowing the use of personal watercraft on common waterways.
    (A) of violating state laws for allowing

    of violating state laws to allow

    (B) that it violates state laws that allowed

    (C) that it violated state laws allowing

    (D) that state laws were being violated allowing

    Show Answer and Explanation
    Click here for the video explanation from our GMAT product.

    Right off the bat, (A) and (B) are problematic. When we use the idiom “on the grounds” we usually say “that” and not “of.” It’s possible to use “of,” but we need a simple noun, like, “He wasn’t allowed in on the grounds age.” (C) uses the correct word “that.” but the rest of the sentence is in past tense, and (C) switches to present “violates.” (E) uses passive voice, which is a huge no-no on the GMAT. It makes sense to say “X violates the law,” but it really isn’t a good idea to say “the law is violated.” Also the progressive “were being” is awkward and not necessary. The answer is (D).
  •  

  • Extracting pure aluminum from bauxite and other ores using the Hall-Héroult process, where a vast amount of electrical energy separates the element from aluminum oxide, making aluminum an energy-dense resource with a huge carbon footprint.
    (A) using the Hall-Héroult process, where a vast amount of electrical energy separates the element from aluminum oxide, making

    (B) by means of the Hall-Héroult process, in which a vast quantity of electrical energy separates the element from aluminum oxide to make

    (C) by means of using the Hall-Héroult process, in which a vast quantity of electrical energy separates the element from aluminum oxide, is making

    (D) by means of the Hall-Héroult process, in which a vast amount of electrical energy separates the element from aluminum oxide, makes

    (E) using the Hall-Héroult process, where a vast amount of electrical energy separates the element from aluminum oxide, makes

    Show Answer and Explanation
    Click here for the video explanation from our GMAT product.

    The sentence begins with the gerund phrase, “Extracting pure aluminum,” and “Extracting” is the subject. (A) and (B) make the missing verb mistake. Every phrase describes the noun but doesn’t have an action, so the sentences are fragments.

    (C) uses the present progressive “is making,” which doesn’t fit here. The rest of the sentence is in the present tense, so we can’t switch to present progressive. The problem with (E) is that it uses “where” to refer to something that isn’t a place. Casual English uses “where” the way that (E) does to refer to a thing. In this case, it’s the process. In formal English, “where” only refers to places. For example, I can say, “New York, where there are many tall buildings…” but I can’t say, “Magoosh, where I get GMAT questions…” because New York is a place and Magoosh is not ? The answer is (D).
  •  

  • At the Battle of Agincourt, the muddy field delayed the advance of the French infantrymen, and this allowed the English longbowmen to be able to inflict significant damage on them, and the English infantry eventually eliminated their reduced numbers easily.
    (A) At the Battle of Agincourt, the muddy field delayed the advance of the French infantrymen, and this allowed the English longbowmen to be able to inflict significant damage on them, and the English infantry eventually eliminated their reduced numbers easily

    (B) ABecause the muddy field delayed the advance of the French infantrymen at the Battle of Agincourt, the English longbowmen could inflict significant damage on them, and, their numbers reduced, the English infantry easily had been able to eliminate them

    (C) The muddy field delayed the advance of the French infantrymen at the Battle of Agincourt, allowing the English longbowmen to inflict significant damage on them, and because the number of the French infantrymen was reduced, the English infantry easily could eliminate those remaining few

    (D) The French infantrymen at the Battle of Agincourt tried to advance but had been delayed by the muddy field, which allowed the English longbowmen to inflict significant damage on them, and the English infantry eventually would eliminate their reduced numbers with ease

    (E) The French infantrymen at the Battle of Agincourt were delayed in their advance by the muddy field, it allowed the English longbowmen to inflict significant damage on them, their numbers were reduced, and the English infantry easily could eliminate those left

    Show Answer and Explanation
    Click here for the video explanation from our GMAT product.

    Here’s another formal English vs. casual English issue. (A) uses the pronoun “this” to refer to an action, the delaying of the French infantryman. Casual English speakers use “this” to refer to anything and everything, but with formal English, we can’t. “This” has to refer to a specific thing that does an action. It can’t refer to an action.

    (B) makes a tense mistake. It’s all past tense (“delayed’), but then we have “had been able,” which would suggest that the final event came before the other events, and that doesn’t make sense. Eliminating them can’t come before the longbowman or the mud, so this doesn’t work. (D) has two problems. First, it also has a tense issue. It says “tried to advance but had been delayed.” This would mean that being delayed happened before the advancing, which doesn’t make sense because we’re told that they were advancing but then got delayed by the mud. The second issue is that it uses “which” to refer to the delaying. Just like with “this,” we can’t use “which” to refer to an action or situation. Yes, we all talk that way, but on the GMAT it’s not allowed.

    Finally, (E) uses “it” to refer to the action in the previous clause, so it makes that same mistake as (A) and (D). This may feel familiar to you from other SC questions that have “laundry list parallelism”— fact #1, fact #2, fact #3, and fact #4. The problem with this design is that we lose all information about how these facts are related logically. It’s not concise and clear, so it’s a no-go.

    The answer is (C).
  •  

  • The vice-president of the engineering firm argued that the biggest advantage of the proposed alloy for the designs of the new fuselage would not lie in its unusually light weight but in its superior resistance to the corrosive influence of the elements.
    (A) would not lie in its

    (B) would lie not in its

    (C) will lie in not their

    (D) will not lie in their

    (E) would lie in not its

    Show Answer and Explanation
    Click here for the video explanation from our GMAT product.

    The first split has to do with the pronoun agreement. The subject is the “proposed alloy.” “Alloy” is singular and (C) and (D) both use “their,” so they’re out.

    The second split concerns the Inside/Outside rule, which you can read about here: Once Outside or Twice Inside. This is a rule that many students don’t see in school, so it’s important to learn it. First start with correlative conjunctions like these:

    Both X and Y

    Neither X nor Y

    Not X but Y

    Not only X but also Y

    Each of these has two individual conjunctions that describe the two halves of some parallelism. Whenever you use these, the parallelism has a clear start at the beginning of the first word. Anything that is within the “both X and Y” structure is “inside,” and anything that comes before or after it is “outside.” If a preposition applies to words in both branches of the parallelism, we have two options:

    Once Outside: to both X and Y

    Twice Inside: both to X and to Y

    These are the ONLY valid options. You’ll commonly see other constructions that are considered errors:

    One inside: both to X and Y

    Once outside, once inside: to both X and to Y

    In this question, we say “not … but …” so where should we put the preposition? The word “lie” has to go once on the outside and the preposition “in” has to appear twice inside. Another way to think about this is that we need “lie not in” so we can get the first “in” inside and “lie” on the outside. The second “in” comes later in the sentence with “but in its superior resistance.” (B) is the only choice to get this right, and it’s the answer. If this rule feels confusing to you, take some time and look carefully at the other answer choices to see what they do and demonstrate to yourself that they don’t follow the rule correctly. There’s also another problem in this post that has an issue with this same rule.

    Choices (A) and D) both put “lie” after the word “not,” so it appears on the inside, which is wrong. (C) and (E) put the preposition “in” before the word “not,” which is once on the outside and also wrong. (B) is the answer.
  •  

  • The ability of neuroscientists to pinpoint the exact locations in the brain where complex sensory responses to a variety of stimuli occur, such as a logic-based board game or a series of violent images, have become ever more precise.
    (A) The ability of neuroscientists to pinpoint the exact locations in the brain where complex sensory responses to a variety of stimuli occur, such as a logic-based board game or a series of violent images, have become ever more precise

    (B) The ability of neuroscientists has become ever more precise in pinpointing the exact locations in the brain where complex sensory responses to a variety of stimuli occur, such as a logic-based board game or a series of violent images

    (C) Neuroscientists have become able to pinpoint ever more precisely the exact locations in the brain where occur complex sensory responses to such a variety of stimuli as a logic-based board game or a series of violent images

    (D) Neuroscientists can pinpoint ever more precisely the exact brain locations where complex sensory responses are to occur, to such a variety of stimuli like a logic-based board game or a series of violent images

    (E) Neuroscientists’ ability to pinpoint the exact brain locations in which complex sensory responses occur to a variety of stimuli as would be a logic-based board game or a series of violent images became ever more precisely so

    Show Answer and Explanation
    Click here for the video explanation from our GMAT product.

    Choice (A) doesn’t work for two reasons. First the subject of the sentence, “the ability of neuroscientists,” is indirect, which is stylistically problematic. Then there’s a subject-verb agreement error because “have become” is plural, and ‘ability’ is singular. They try to hide the subject-verb agreement error by putting a bunch of modifiers in between the subject and verb, which is why it’s always a good idea to check all nouns and verbs every time you read a sentence ?

    Choice (B) tries to hide an error as well. The words ‘able’ and ‘ability’ have to go with an infinitive. An infinitive is a verb in a form like “to run” or “to see.” Here the preposition is “in” which is idiomatically incorrect.

    On to choice (D)! The phrase “where complex sensory responses are to occur” is a problem. This phrasing makes it sound like there is some cosmic plan that these responses must occur. This doesn’t fit with the natural science context, so it’s wrong. As you can see, the hardest SC problems have extremely subtle errors in them.

    Choice (E) is similar to (A) in many ways. It has a strange subject (“Neuroscientists’ ability” is clunky), the verb is super far away from the subject, and the verb has an error. In this case, “became” is past tense and the rest of the sentence is in present.

    Choice (C) is the answer.
  •  

  • Dante Rossetti and his colleagues, in calling their group the Pre-Raphaelite Brotherhood, sought a return to the classical ideals of painting that held sway before Raffaello, to what governed the work of 15th century artists such as Sandro Botticelli.
    (A) Raffaello, to what governed the work of 15th century artists such as Sandro Botticelli

    (B) Raffaello, artistic principles that would govern the work of 15th century artists such as Sandro Botticelli

    (C) Raffaello who governed the work of 15th century artists like Sandro Botticelli

    (D) Raffaello by which the work of 15th century artists such as Sandro Botticelli was governed

    (E) Raffaello that had governed the work of 15th century artists like Sandro Botticelli

    Show Answer and Explanation
    Click here for the video explanation from our GMAT product.

    Because Choice (A) is the answer, let’s skip it for now and come back to it at the end. Choice (B) uses “would govern” which doesn’t fit here. “Would govern” in this context refers to what could be in the future from the perspective of the past, which is not what we’re doing.

    Choice (C) changes the meaning of the sentence. The ideals governed the 15th century artists. (C) makes it sound like Raffaello himself governed the artists, which doesn’t make any sense. This choice also uses ‘like’ to introduce an example, which isn’t allowed on the GMAT. Choice (D) has the same problem. There’s no comma, so it still sounds like we’re saying that Raffaello did the governing. (E) is exactly the same. It sounds like Raffaello was the governing force, and it uses ‘like’ for an example.

    Choice (A) is the correct answer. “Classical ideals” is a noun, and “what governed the work of 15th century artists” is a substantive clause, so that’s legitimate for parallelism. None of the other four answer choices execute the parallel structure correctly.
  •  

  • One of Hannibal’s successful battle techniques was charging wild elephants directly at the Roman armies, trampling infantry and inducing disarray in the ranks, but Scipio’s successful counterstrategy was to command that his men should part, creating channels that allowed the elephants simply to pass through, with them killing the elephants behind his ranks.
    (A) to command that his men should part, creating channels that allowed the elephants simply to pass through, with them killing

    (B) to command his men to part, thereby creating channels that allow the elephants simply to pass through, and then to kill

    (C) commanding his men to part, creating channels allowing the elephants simply to pass through, and then to kill

    (D) commanding that his men should part, creating channels to allow the elephants simply to pass through, and then killing

    (E) commanding his men to part, creating channels allowing the elephants simply to pass through, and then having them kill

    Show Answer and Explanation
    Click here for the video explanation from our GMAT product.

    This one is tricky because it involves parallelism at a couple of different levels. First of all, we are comparing Hannibal’s vs. Scipio’s approaches. Hannibal’s technique was “charging”, so Scipio’s counterstrategy has to be parallel to this. To match “charing” is has to be “commanding”, not “to command.” It’s important to note that if it weren’t for parallelism, the use of the infinitive in this context is completely allowed.

    After the word command/commanding, the most natural idiom is an infinitive: to command B to do X. The “that”-clause structure here, “that his men should part”, is very awkward. It does sound like fancy or old-fashioned English, which is usually a good sign that something is wrong. Reject (A) & (D).

    The structure “with them killing” is 100% wrong. In general, the GMAT hates “with” + [noun] + [participle] to encapsulate an action. If we want to talk about an action, we need to use a fully bonafide verb. Another reason to reject (A).

    Scipio is commanding his men to do two things: (1) to part, and (2) to kill the elephants. These two actions need to be in parallel as well. Furthermore, the correct idiom is “command” + [infinitive], while “command” + [gerund] is 100% wrong. Choice (C) correctly has two infinitives, “to part” and “to kill”. Choice (C) is the only answer that has correct parallelism in both places, so this is the only possible answer.

    Both (D) and (E) mix up the two things that Scipio commanded the men to do. Losing that distinction changes the meaning of the sentence. Remember, parallelism is not automatically correct especially when it entails a major change in meaning.
  •  

  • If medical researchers are correct, then the human microbiome, made up of the microorganisms in our body, may hold the cure to diseases that have long plagued humanity, amounting to a major oversight in Western medicine that has, until recently, all but ignored any such role of the microbiome.
    (A) humanity, amounting to a major oversight in Western medicine that has, until recently, all but

    (B) humanity, a discovery that is amounting to a major oversight in Western medicine, which, until recently, had all but

    (C) humanity, a discovery amounting to a major oversight by Western medicine: until recently, it all but

    (D) humanity, amounting to a major oversight made by Western medicine, one that, until recently, all but

    (E) humanity, which amounts to a major oversight in Western medicine and until recently it all but

    Show Answer and Explanation
    Click here for the video explanation from our GMAT product.

    On the hardest SC questions, you may face a summative modifier, which is a word that ‘encapsulates’ the action of the preceding clause. In this case, the word “amounting” could refer to microbiome, diseases, or humanity. Which of these three is the author of this sentence arguing accounts for the oversight?

    To do this properly, we need a word that encapsulates or captures the preceding phrase. Something that will wrap it up and refer to all three at once. This is called a summative modifier. It sums up what was just said. Luckily, the GMAT will never give you more than one summative modifier and build the question out of your picking the correct one. As you can see, “discovery” is the summative modifier. It works well and it’s the only one.

    This is such an important insight because right away we can eliminate (A), (D), and (E). Any form of “amounts” is fundamentally ambiguous, and therefore a violation of the style rules. All we have to do now is compare (B) and (C) and figure out which is best (or least wrong, as is often the case).

    In (B), the word “which” refers to the phrase “a major oversight.” We know this because when “which” is used, it refers to the subject of the noun phrase that comes right before it. (B) is making an error, then, because the major oversight is not the thing that is doing the ignoring the role of the microbiome. The oversight is ignoring the microbiome. (C) states this correctly. Western medicine is what has been ignoring the microbiome. That is the subject that needs to be described as making an oversight. That’s why (C) is the answer.
  •  

  • The FDA enacted these recent restrictions both to prohibit individual physicians from forming financial partnerships with pharmaceutical companies and to forbid the companies to advertise directly to the physicians.
    (A) both to prohibit individual physicians from forming financial partnerships with pharmaceutical companies and to forbid the companies to advertise

    (B) both to prohibit individual physicians to form financial partnerships with pharmaceutical companies while forbidding the companies to advertise

    (C) to both prohibit individual physicians from forming financial partnerships with pharmaceutical companies and to forbid the companies from advertising

    (D) both to prohibit individual physicians from forming financial partnerships with pharmaceutical companies and to forbid the companies from advertising

    (E) to prohibit both individual physicians to form financial partnerships with pharmaceutical companies and to forbid the companies from advertising

    Show Answer and Explanation
    Click here for the video explanation from our GMAT product.

    This sentence has two similar verbs that form two completely different idioms. The verb “to prohibit” always takes “from” + [gerund]; using the infinitive is 100% incorrect with this verb. By contrast, the verb “to forbid” always takes the infinitive; using “from” + [gerund] is 100% incorrect with this verb.

    Choice (A): “prohibit … from … forbid … to” = correct

    Choice (B): “prohibit … to … forbid … to” = idiom mistake

    Choice (C): “prohibit … from … forbid … from” = idiom mistake

    Choice (D): “prohibit … from … forbid … from” = idiom mistake

    Choice (E): “prohibit … to … forbid … from” = double-whammy idiom mistake

    This is all we need to identify the correct answer (A). There are also other issues at play here, so we’ll talk about them too. One important rule that we’ve discussed elsewhere on this post is the Once Outside, Twice Inside rule. This rule comes into effect when we have correlated conjunctions—that is, a pair of conjunctions that mark the two halves of the parallelism: examples include “both X and Y,” “neither X nor Y,” “not X but Y,” and “not only X but also Y.” When any of these are used, the parallelism has a clear start at the beginning of the first word. Anything that is within the “both X and Y” structure is “inside,” and anything that comes before or after it is “outside.” If a preposition applies to both words, there are two ways we can go:

    Once Outside: to both X and Y

    Twice Inside: both to X and to Y

    In this question, the parallel elements are the infinitive “to prohibit … and to forbid.” Thus, we could have:

    Once Outside: to both prohibit and forbid

    Twice Inside: both to prohibit and to forbid

    The incorrect patterns are

    Once inside: both to prohibit and forbid*

    Once outside once inside: to both prohibit and to forbid

    Choice (A): twice inside = correct

    Choice (B): twice inside = correct

    Choice (C): once outside, once inside = incorrect

    Choice (D): twice inside = correct

    Choice (E): once outside, once inside = incorrect

    Finally, there is one more error in this problem that is good to notice. The “both X and Y” structure is a standard parallelism frame. In this structure, X and Y need to be in parallel: two infinitives. Choice (B) violates the parallelism: “both to prohibit … while forbidding.” Choice (B) is incorrect.

    For all of these reasons, the only possible answer is (A).

The post GMAT Sentence Correction with 20 Practice Questions and Answers appeared first on Magoosh GMAT Blog.
This Blog post was imported into the forum automatically. We hope you found it helpful. Please use the Kudos button if you did, or please PM/DM me if you found it disruptive and I will take care of it. -BB
Magoosh GMAT Instructor
Joined: 28 Dec 2011
Posts: 4452
Own Kudos [?]: 28605 [0]
Given Kudos: 130
Integrated Reasoning on the GMAT: The Complete Guide [#permalink]
Expert Reply
FROM Magoosh Blog: Integrated Reasoning on the GMAT: The Complete Guide


If you’re sitting down to your first GMAT Integrated Reasoning section, you might not know what to think! You’ll have 30 minutes to answer 12 Integrated Reasoning GMAT questions involving a ton of data analysis, critical thinking, and quantitative reasoning. But by preparing early—and thoroughly!—you’ll be able to get the Integrated Reasoning GMAT score that you want on test day.

What is GMAT Integrated Reasoning?
The GMAT Integrated Reasoning section tests your quant-based and verbal-based reasoning in 4 parts: multi-source reasoning, table analysis, two-part analysis, and graphics interpretation. Essentially, the GMAT IR section is testing for your data analysis skills.

The big question for many students: just what is the GMAT IR score importance? IR is relatively important (and increasingly so!), so it’s vital not to ignore it!

Overall, you can expect to see 12 questions in 30 minutes. It’s not quite as simple as it sounds, though: each graph or prompt will have multiple questions. In addition, you’ll have to answer a question before you can move on—and you can’t go back to a question once you’ve answered it.



Check out this video of Magoosh’s Introduction to Integrated Reasoning lesson!

Integrated Reasoning Question Types on the GMAT
Multi-Source Reasoning
Multi-source reasoning questions show you a split screen: on the left, you’ll have three clickable cards, each with a piece of information that will help you answer a particular question, and which you can only see one of at a time. The questions are either standard five-choice multiple choice or multiple dichotomous choice. You’ll have two answer choices (e.g. “true/false”) for each part of a three-part question.

For more sample multi-source reasoning problems, click here!

Table Analysis
Table analysis questions give you a sortable table of numbers. These are accompanied by multiple dichotomous choice questions, in which you have two answer choices (e.g. “true/false”) for each part of a three-part question.

Check out a sample table analysis problem!

Graphics Interpretation
For graphics interpretation questions, you’ll receive some visual information in the form of a chart or a graph, then questions containing two drop-down menus each. These menus will have you fill in blanks within a sentence according to the data shown in the visual.

Check out more sample graphics interpretation problems!

Two-Part Analysis
Two-part analysis questions give you a large prompt, followed by a question-and-answer table. You will fill out the answers for each of two questions, which can vary; they may be partially or completely related, but they will always be interdependent.

Check out a sample two-part analysis problem!

Strategies for Getting a Good Integrated Reasoning GMAT Score
Getting a good Integrated Reasoning GMAT score can be tricky—not least of all because a lot of people aren’t sure what a good IR score is! Unlike the total GMAT score, which is scored between 200 and 800, IR is scored between 1 and 8. Most people would consider a good score to be above the 50th percentile—in other words, better than half of test-takers’ scores. For IR, this is a 5 or higher.

So how do you get that good score of 5+ on GMAT IR? Here are a few keys to succeeding:

  • Know the question formats by heart. IR questions can be—and in fact, are meant to be—overwhelming. By knowing the formats in advance, you’ll be setting yourself up for success. That’s one part of the section that won’t be overwhelming!
  • Prepare yourself for strict time management. You get no partial credit for a question in IR. Furthermore, each of the 12 “questions” has multiple parts. So if you miss one part of a three-part question, you won’t get any points for that question at all. While 30 minutes for 12 questions sounds like a lot, 30 minutes for what can be 36 questions or more ends up being a whole lot less. Careful prep with timed practice is key!
  • Get into an executive mindset. Executive function, that is, not business executive. These skills involve things like choosing priorities, evaluating benefits and liabilities, and coming up with strategies. Practice this by reading materials like The Wall Street Journal and The Economist to read and review different types of analyses.
  • Work on your visual literacy. Charts and graphs are extremely important in GMAT IR! Again, The Wall Street Journal and The Economist can be incredibly helpful here. Make sure that you work not only on breaking down the parts of a graph, but also on understanding and analyzing them in context.
GMAT Integrated Reasoning Practice
Need Integrated Reasoning GMAT practice? You’re not alone! That’s why Magoosh is excited to present you with the ultimate GMAT Integrated Reasoning eBook: Magoosh’s Complete Guide to GMAT Integrated Reasoning! Here, you’ll find detailed explanations and tips for each IR question type, as well as practice witin each area.





Still want more? You can find Magoosh’s guide to the official GMAT Integrated Reasoning practice here!

A Final Note
GMAT Integrated Reasoning questions are designed to throw a lot of data at you, fast. And while a lot of test-takers will let that throw them off their game, you can ensure that you’re all set for test day by familiarizing yourself with these question types and practicing, practicing, practicing! The more used to the question types you are, the easier your test day experience will be. And the GMAT IR prep tools in this post will help you get there. Good luck!

The post Integrated Reasoning on the GMAT: The Complete Guide appeared first on Magoosh GMAT Blog.
This Blog post was imported into the forum automatically. We hope you found it helpful. Please use the Kudos button if you did, or please PM/DM me if you found it disruptive and I will take care of it. -BB
Magoosh GMAT Instructor
Joined: 28 Dec 2011
Posts: 4452
Own Kudos [?]: 28605 [0]
Given Kudos: 130
GMAT Word Problems: Introduction, Strategies, and Practice Questions [#permalink]
Expert Reply
FROM Magoosh Blog: GMAT Word Problems: Introduction, Strategies, and Practice Questions


You may love GMAT word problems or you may hate them, but you can’t get around them if you want to ace the GMAT Quant section. No matter what your feelings are about this problem type, though, Magoosh’s experts have put together everything you need to know (and practice!) GMAT word problems in order to master them before test day.

What to Expect from GMAT Word Problems[/*]
[*]Strategy Guide: What’s the Trick to Mastering GMAT Word Problems?
[*]GMAT Word Problem Practice Questions[*]A Final Word on GMAT Word Problems[/list]
GMAT Quant section. How much of the GMAT is word problems? Within the Quant section, actually a whole lot! A study of official GMAT questions from actual tests show that word problems account for 58.2% of all GMAT math questions.

 



In other words, test-takers should anticipate a word problem cropping up (on average) in three out of every five questions you’ll see in Quant. Because of GMAT word problems’ prevalence, you can expect to see both Data Sufficiency and Problem Solving questions in this format. The question format and answer choices may look different, but the basic premise will be the same.

You may be feeling the pressure, but hang in there! If you’re worried about how to master word problems on the GMAT, keep reading for our GMAT Word Problems strategy guide.



Variables in GMAT Answer Choices: 2 Approaches.

tips for plugging in numbers that you should use!

Here’s a quick summary of how to quick the best numbers for a particular problem:

  • Remember that the GMAT has a broad definition of “number” that goes beyond positive integers! Zero, fractions, and negatives are all included. Work on developing number sense to help select the best numbers in a given scenario.
  • For percent problems, think outside the box: GMAT test writers know lots of students pick 100. Try 500 or 1000 instead.
  • Don’t try to pick numbers for questions involving more than one percent increase or decrease.
  • Pay attention to units and convert them appropriately. This is particularly important in solutions and mixing problems!
  • Don’t pick 1 as a number—it has too many unique properties.
A separate case involving plugging in, rather than picking, numbers: When all the answer choices are numerical, one further strategy we have at our disposal is backsolving. Using this strategy, we can pick one answer, plug it into the problem, and see whether it works. If this choice is too big or too small, it guides us in what other answer choices to eliminate. Typically, we would start with answer choice (C), but if another answer choice is a particularly convenient choice, then we would start there.



Click here for a video answer and explanation to GMAT Word Problem 1![/b]

Click here for a text answer and explanation to GMAT Word Problem 1!
Our task is to determine the ratio of Bob’s trees to Ann’s trees. Let’s label these numbers of trees with variables:

Bob’s trees→B, Ann’s trees→A

With these variables, we can express the ratio we want to determine:

\(B/A\) =?

Statement 1:

Ann planted 20 trees more than Bob planted.

Let’s translate this into an equation using A and B:

\( A=B+20 \)
Now we can substitute this into our ratio, replacing A:

\(B/A\) = \( B/(B+20) \)

No matter what simplifications we make, we cannot find a numerical value for this fraction. We would need a value for B. We cannot determine the ratio. Statement 1 by itself is not sufficient.

Statement 2:

Ann planted 10 percent more trees than Bob planted.

Let’s translate this into an equation using A and B:

\(A=1.10 x B \)
Again, let’s substitute this in for A in our ratio:

\(B/A\) = \( B/(1.10B) \)

= \(1/1.1 \)

We found a value for the ratio of Bob’s trees to Ann’s trees. Statement 2 alone is sufficient.
 

[*]The Townville museum was open for 7 consecutive days. If the number of visitors each day was 3 greater than the previous day, how many visitors were there on the first day?

(1) There were a total of 126 visitors for the 7 days.

(2) The number of visitors on the seventh day was three times the number of visitors on the first day.
[/list]
A. Statement 1 ALONE is sufficient to answer the question, but statement 2 alone is NOT sufficient.

B. Statement 2 ALONE is sufficient to answer the question, but statement 1 alone is NOT sufficient.

C. BOTH statements 1 and 2 TOGETHER are sufficient to answer the question, but NEITHER statement ALONE is sufficient.

D. Each statement ALONE is sufficient to answer the question.

E. Statement 1 and 2 TOGETHER are NOT sufficient to answer the question.

Click here for a video answer and explanation to GMAT Word Problem 2!

Click here for a text answer and explanation to GMAT Word Problem 2!
If x is the number of visitors on the first day, then:

x = # of visitors on the 1st day

x + 3 = # of visitors on the 2nd day

x + 6 = # of visitors on the 3rd day

x + 9 = # of visitors on the 4th day

x + 12 = # of visitors on the 5th day

x + 15 = # of visitors on the 6th day

x + 18 = # of visitors on the 7th day

1) Adding up the number of visitors gives us:

x + (x + 3) + (x + 9) + (x + 12) + (x + 15) + (x + 18) = 126

We could simplify and solve this for x. So Statement 1 is sufficient.

2) x + 18 = 3x

Again, we can simplify this and solve for x. So Statement 2 is sufficient.

Answer: (D)
 

[*]Two teachers, Ms. Ames and Mr. Betancourt, each had N cookies. Ms. Ames was able to give the same number of cookies to each one of her 24 students, with none left over. Mr. Betancourt was also able to give the same number of cookies to each one of his 18 students, with none left over. If N > 0, what is the value of N?

(1) N<100

(2) N > 50
[/list]
A. Statement 1 ALONE is sufficient to answer the question, but statement 2 alone is NOT sufficient.

B. Statement 2 ALONE is sufficient to answer the question, but statement 1 alone is NOT sufficient.

C. BOTH statements 1 and 2 TOGETHER are sufficient to answer the question, but NEITHER statement ALONE is sufficient.

D. Each statement ALONE is sufficient to answer the question.

E. Statement 1 and 2 TOGETHER are NOT sufficient to answer the question.

Click here for a video answer and explanation to GMAT Word Problem 3!

Click here for a text answer and explanation to GMAT Word Problem 3!
This question is really about common multiples and the LCM (note that it is different than finding the set of all multiples, though!). If Ms. Ames can give each of her 24 students k cookies, so that they all get the same and none are left over, then 24k = N. Similarly, in Mr. Betancourt’s class, 18s = N.

What are the common multiples of 18 and 24?

18 = 2×9 = 2×3×3 = 6×3

24 = 3×8 = 2×2×2×3 = 6×4

From the prime factorizations, we see that GCF = 6, so the LCM is

LCM = 6×3×4 = 72

and all other common multiples of 18 and 24 are the multiples of 72: {72, 144, 216, 288, 360, …}

Statement #1: if N<100, the only possibility is N = 72. This statement, alone and by itself, is sufficient.

Statement #2: if N > 50, then N could be 72, or 144, or 216, or etc. Many possibilities. This statement, alone and by itself, is not sufficient.

Answer = (A)
 

[*]A certain zoo has mammals and reptiles and birds, and no other animals. The ratio of mammals to reptiles to birds is 11:8:5. How many birds are in the zoo?

(1) there are twelve more mammals in the zoo than there are reptiles

(2) if the zoo acquired 16 more mammals, the ratio of mammals to birds would be 3:1
[/list]
A. Statement 1 ALONE is sufficient to answer the question, but statement 2 alone is NOT sufficient.

B. Statement 2 ALONE is sufficient to answer the question, but statement 1 alone is NOT sufficient.

C. BOTH statements 1 and 2 TOGETHER are sufficient to answer the question, but NEITHER statement ALONE is sufficient.

D. Each statement ALONE is sufficient to answer the question.

E. Statement 1 and 2 TOGETHER are NOT sufficient to answer the question.

Click here for a video answer and explanation to GMAT Word Problem 4!

Click here for a text answer and explanation to GMAT Word Problem 4!
A short way to do this problem. The prompt gives us ratio information. Each statement gives use some kind of count information, so each must be sufficient on its own. From that alone, we can conclude: answer = D. This is all we have to do for Data Sufficiency.

Here are the details, if you would like to see them.

Statement (1): there are twelve more mammals in the zoo than there are reptiles

From the ratio in the prompt, we know mammals are 11 “parts” and reptiles are 8 “parts”, so mammals have three more “parts” than do reptiles. If this difference of three “parts” consists of 12 mammals, that must mean there are four animals in each “part.” We have five bird “parts”, and if each counts as four animals, that’s 5*4 = 20 birds. This statement, alone and by itself, is sufficient.

Statement (2): if the zoo acquired 16 more mammals, the ratio of mammals to birds would be 3:1

Let’s say there are x animals in a “part”—this means there are currently 11x mammals and 5x birds. Suppose we add 16 mammals. Then the ratio of (11x + 16) mammals to 5x birds is 3:1.

(11x + 16)/(5x) = 3/1 = 3

11x + 16 = 3*(5x) = 15x

16 = 15x – 11x

16 = 4x

4 = x

So there are four animals in a “part”. The birds have five parts, 5x, so that’s 20 birds. This statement, alone and by itself, is sufficient.

Both statements are sufficient. Answer = D.
 



average speed to total distance traveled, from total time to total amount.

The key now is to put them into practice. Jot down these techniques or bookmark this post so you can come back as you continue your practice with GMAT word problems. You can also check out our posts on compound interest and Venn diagrams for more practice with GMAT word problems. Good luck!

This post was written with contributions from our Magoosh content creator, Rachel Kapelke-Dale.

The post GMAT Word Problems: Introduction, Strategies, and Practice Questions appeared first on Magoosh GMAT Blog.
This Blog post was imported into the forum automatically. We hope you found it helpful. Please use the Kudos button if you did, or please PM/DM me if you found it disruptive and I will take care of it. -BB
Magoosh GMAT Instructor
Joined: 28 Dec 2011
Posts: 4452
Own Kudos [?]: 28605 [0]
Given Kudos: 130
GMAT Error Log: The Key to GMAT Success (Free Template Included) [#permalink]
Expert Reply
FROM Magoosh Blog: GMAT Error Log: The Key to GMAT Success (Free Template Included)


Your GMAT journey will be both challenging and rewarding. You will have highs and lows. There will be days that will build your confidence and then other days you will want to forget about completely. We have all had those days: you are studying and you feel like you can’t get any question right. But before you crumble up that paper, wipe clean that dry erase board, or power down that computer, STOP! Document your struggles and capture that data!

Why Keeping an Error Log is So Important
Our mistakes are chock full of information about our test taking abilities, strategies, and more importantly, our habits. Whether you’re GMAT studying for the first time or preparing for a retake, a critical look at how we approach, solve, and check our work will not only direct us to which areas we should study next, but will make us conscious about the bad habits we exhibit every time we attempt a GMAT problem.



We are always told to review our questions after we complete practice problems and the best way to advance your skills is by using a GMAT Error Log!

I know what you’re thinking: not another GMAT wonder tool you have to buy to achieve your goal score.

We get it. The GMAT is not only taxing on you mentally, physically, and socially (no one wants to hang out with the guy with Critical Reasoning flashcards at the party), but the GMAT is also COSTLY! That’s the beauty of the GMAT Error Log—it’s a completely free tool! Even better, keep reading and I will share with you a template I use to save you time on finding one online.

Yes, the power of self evaluating your GMAT progress can be yours with a few clicks of a mouse!

How to Use an Error Log
GMAT prep is broken down into three parts: Content, Problem Solving, & Testing.

What I have seen that keeps test-takers from earning their top score is that they haven’t spent enough time in the Problem Solving stage. Test-takers would read 100% of the material and complete 4 to 5 practice CAT exams, but would only successfully complete 20-30% of all the questions on the Magoosh platform and even less when it comes to the Official Guide.

Knowing the ratios of a 30-60-90 triangle, how to FOIL, or what is a noun is important, but you must drill these topics in various various ways to really understand these concepts and how to answer these questions on the exam.

Taking CAT exams in isolation will not improve your score. It is in careful review and reflection is where you will develop skills and earn points. CAT exams are a major time investment and are extremely valuable, so you don’t want to waste too much time to only answer 30 or more random questions.

What I have seen successful test-takers do is create mini problems sets (5, 8, 10, or 16 questions) and use those as a sample test to gauge their skill set. In these practice problems is where the GMAT Error Log shines!



Let’s say each day for a week, you complete 10 to 20 random problems of various question types and difficulties. You are bound to miss a few questions. What most people do when they get a question wrong is read the explanation, say “oh I got it,” and move on.

Here is where you leverage the GMAT Error Log. Not only will you read the explanation, you will also input the question you got wrong in the Error Log, what you did wrong, and what you will do differently next time.

This seems like a small task today, but in the beginning, it can feel very daunting. After about 20 or so questions, though, you will get into the habit of updating your error log and begin to see the fruits of your labor.

Leading up to a test day, a great Error Log can be a great review tool right before a CAT exam.

When I work with my test-takers I teach them how to build a detailed Error Log with great care because it is those careless errors or habits that are written in the Error Log that jumps your GMAT score from a 620 to 650 or 690 to that 700.

How to Structure Your GMAT Log
Your Error Log does not need to have many bells and whistles. It doesn’t need to be something with macros and automated graphic generators. It doesn’t have to be a spreadsheet or digital at all. I actually keep two logs – an old school black and white marble notebook and a digital copy.

The Error Log just has to fit your method of note-taking and meet your dedication to the process.

Error Logs lose value when they are not updated. You only get out of the Error Log what you put in. I can never guarantee your score to improve, but I can say that having a running list of all your mistakes can help prevent you from making them again.

Here is the key information you want to have in any GMAT Error Log:

  • Write down the source and question number, type of problem, and the concepts being asked in the problem. These are your basic details that will help you notice patterns
  • Next, write answers to some key questions:
    • Why did you miss the question?
    • Why was your answer wrong?
    • Why was the correct answer correct?
    • What will you do to avoid this next time around?

This sounds extremely simple, because it is – the challenging part is sticking to the habit.

After a few weeks, you will have an error log that you will cherish and will be excited to fill it up. So go out there and make some GMAT mistakes! I am here to help you—feel free to comment below and include a copy of your own error log and I will provide you some feedback and additional examples.

The post GMAT Error Log: The Key to GMAT Success (Free Template Included) appeared first on Magoosh GMAT Blog.
This Blog post was imported into the forum automatically. We hope you found it helpful. Please use the Kudos button if you did, or please PM/DM me if you found it disruptive and I will take care of it. -BB
Magoosh GMAT Instructor
Joined: 28 Dec 2011
Posts: 4452
Own Kudos [?]: 28605 [0]
Given Kudos: 130
Build Your Own GMAT Study Schedule [#permalink]
Expert Reply
FROM Magoosh Blog: Build Your Own GMAT Study Schedule


You may have heard that studying for the GMAT is similar to preparing for a marathon. Not many people could run 26.2 miles (42.195 kilometers) without months of preparation! Though you probably won’t be as sweaty as you would be if you were to run for hours, you’ll still need endurance, speed, and strong mental skills to get through the GMAT with the score you deserve. Now, if you’re planning to run a marathon, your schedule is easy to fill out — It’s running on Monday, Tuesday probably some running, and then more running on Wednesday, Thursday, and so on. Not so simple with the GMAT! On your big day, you need to be prepared to use dozens of different math and grammar rules, track the meaning of several thousand words of text, and solve math problems with no calculator, all while going really fast and not making any mistakes. So more like a three-legged hot yoga CrossFit marathon. So what’s your plan? If you’re looking for some help deciding what to do each Monday, Tuesday, and Wednesday to be ready for the unique challenges of the GMAT, you’ve found it!

Once you have a goal in mind, and you’ve gathered some helpful resources, it’s time to start building your own GMAT study schedule.

Stay motivated — know your goals
What are your hopes and dreams? Who are you and who do you aspire to be? Sorry, I didn’t mean to get too personal, but some reflecting on your reasons for studying for a 3+ hour exam that costs hundreds of dollars and determines a big part of your future is a great place to start!

If you need some ideas, consider your short term vs. your long term goals. Your short term goal might be to take the GMAT in 3 months and score 750, but your long term goal might be to get into a top MBA program and then conquer the world of finance (or maybe just prove to your old calculus professor that you are worth more than the Σ of your parts). I recommend writing down your goals and keeping them somewhere visible — on the fridge, the mirror, or out on your desk. When you’re feeling discouraged or especially tired, look back to your goals to reinvigorate you and remind you of why you’re putting yourself through this.

Before you get started
Before you start building your own GMAT study schedule, you’ll need to get squared away with a couple initial steps. The schedule builder will use a little info to guide you to a recommended study schedule template. Most importantly, it wants to know the results from your diagnostic tests. If you haven’t had all that fun yet, take about an hour and complete both the quantitative and verbal diagnostic tests. Then head back here and you’ll be ready to go.

Build your custom GMAT study schedule
After you’ve completed your diagnostic, just take this brief quiz and then we’ll recommend a template of your study schedule to get started with!

powered by Typeform

The post Build Your Own GMAT Study Schedule appeared first on Magoosh GMAT Blog.
This Blog post was imported into the forum automatically. We hope you found it helpful. Please use the Kudos button if you did, or please PM/DM me if you found it disruptive and I will take care of it. -BB
Magoosh GMAT Instructor
Joined: 28 Dec 2011
Posts: 4452
Own Kudos [?]: 28605 [0]
Given Kudos: 130
How to Solve GMAT Motion Problems [#permalink]
Expert Reply
FROM Magoosh Blog: How to Solve GMAT Motion Problems


Word Problems make up a majority of the quantitative section of the GMAT (almost 60 percent). Of the word problems they’ll face, students tend to need the most help with GMAT motion problems. This type of problem centers around the “dust” formula, which is short for Distance equals Speed multiplied by Time, or \(\text{D}\times\text{S}=\text{T}\). But there are many varieties of motion problem, and we will discuss techniques for each of them. At the end of this article, you’ll also find motion word problems with solutions for you to test your knowledge!

The Distance Equation[/*]
[*]Multi-Segment Motion Problems[/*]
[*]Average Speed[/*]
[*]Multiple Traveler Questions[/*]
[*]Shrinking and Expanding Gaps[/*]
[*]Data Sufficiency[/*]
[*]GMAT Motion Problems Review: Practice Problems[/*]
[/list]
Distance equals Speed multiplied by Time, or \(\text{D}\times\text{S}=\text{T}\). If you learn this basic equation well, you’ll be able to dust your math troubles away! (Insert rimshot.)

 

We can rearrange this formula to determine that Speed is equal to Distance divided by Time, and that Time is equal to Distance divided by Speed. Distance is the measurement of how far apart objects, people, or points are. Speed is the rate at which someone or something is traveling. Time is how long it takes to travel.

 

Let’s demonstrate this. Walking at a constant rate of 160 meters per hour, Monroe can cross a bridge in 2 hours. What is the length of the bridge?

 

Here, the length of the bridge is the distance Monroe must cross. Using Distance equals Speed multiplied by Time, we get:

 

\((160\;\text{meters per hour})\times(2\;\text{hours})=320\;\text{meters}\)

 

Seems simple enough so far, right? Let’s check out a few more GMAT motion problems.





Speed is equal to Distance divided by Time. Let’s take that a step further and talk about average speed. Average speed is defined as total distance traveled divided by the total time period spent traveling. This means that if you have a trip with multiple segments, you’ll want to take the sum of the distances of each segment and divide that by the sum of the times of each segment. Average speed captures the constant speed needed to travel the total distance in the total time.

 

Let’s demonstrate this. Koki drove 16 miles in 10 minutes, and then drove an additional 6 miles in 5 minutes. What is Koki’s average speed for the entire trip in miles per hour?

Click here for the answer and explanation

Well, average speed is the total distance divided by the total time.

 

\(\text{D}_\text{Total}=16\;\text{miles}+6\;\text{miles}=22\;\text{miles}\)

 

\(\text{T}_\text{Total}=10\;\text{minutes}+5\;\text{minutes}=15\;\text{minutes}\)

 

\(\text{S}_\text{Average}=\frac{\text{D}_\text{Total}}{\text{T}_\text{Total}}\)

 

\(\text{S}_\text{Average}=\frac{22\;\text{miles}}{15\;\text{minutes}}\times\frac{60\;\text{minutes}}{1\;\text{hour}}=88\;\text{miles per hour}\)

 

That’s straightforward enough, but what if we are not given any distances or times? It is possible to solve an average speed problem, even if all you are given are the different speeds in each segment of the trip. You might then think that average speed would just be the average of all of the speeds, but that is not correct.

 

Let’s say that Nathaniel drove from Gwenville to Samton at an average speed of 24 miles per hour. He then drove the same route on the return trip back from Samton to Gwenville at an average speed of 36 miles per hour. If you were asked to find Nathaniel’s average speed, it would not just be 30 miles per hour (the average of 24 and 36).

Click here to work through this problem

To see this, let’s go back to our MVP dust formula. Since there are two legs of the trip, we will have two equations. D1, S1, T1; D2, S2, T2. Because Nathaniel’s trip is a round trip, we can assume that D1and D2 are the same, so we will set both of them equal to D.

 

\(\text{D}_1=\text{D}\\\text{S}_1=24\;\text{miles per hour}\\\text{T}_1=\frac{D}{24\;\text{miles per hour}}\)

 

\(\text{D}_2=\text{D}\\\text{S}_2=36\;\text{miles per hour}\\\text{T}_2=\frac{D}{36\;\text{miles per hour}}\)

 

\(\text{D}_\text{Total}=\text{2D}\)

 

\(\text{T}_\text{Total}=\frac{D}{24\;\text{miles per hour}}+\frac{D}{36\;\text{miles per hour}}\)

 

\(\text{S}_\text{Average}=\frac{\text{D}_\text{Total}}{\text{T}_\text{Total}}\)

 

\(\text{S}_\text{Average}=\frac{\text{2D}}{\frac{D}{24\;\text{miles per hour}}+\frac{D}{36\;\text{miles per hour}}}\)

 

We can factor a D out of this fraction.

 

\(\text{S}_\text{Average}=\frac{2}{\frac{1}{24}+\frac{1}{36}}\;\text{miles per hour}\)

 

We can find a common denominator between 24 and 36.

 

\(\text{S}_\text{Average}=\frac{2}{\frac{3}{72}+\frac{2}{72}}\;\text{miles per hour}\)

 

\(\text{S}_\text{Average}=\frac{2}{\frac{5}{72}}\;\text{miles per hour}=\frac{2}{1}\times\frac{72}{5}=\frac{144}{5}\;\text{miles per hour}=28.8\;\text{miles per hour}\)

 

In summary, whenever you want to find the average speed of a round trip, and you are given the two segment speeds, you can put it in the form \(\text{S}_\text{Average}=\frac{\text{2}}{\frac{1}{\text{S}_1}+\frac{1}{\text{S}_2}}\). You can also use this formula to find one of the segment speeds, given the other segment speed and the average speed.





diagram.

 

Let’s say that a car and truck are moving in the same direction on the same highway. The truck is moving at 50 miles an hour, and the car is traveling at a constant speed. At 3 pm, the car is 30 miles behind the truck and at 4:30 pm, the car overtakes and passes the truck. What is the speed of the car?

Click here for the answer and explanation

The car and truck are moving in the same direction, and the car is gaining on the truck. This means that the gap between the vehicles is shrinking and that the gap rate is the difference of the two vehicles’ respective speeds.

 

\(\text{S}_\text{G}=\text{S}_\text{C}-\text{S}_\text{T}\)

 

The distance of the gap is initially 30 miles.

 

\(\text{D}=30\;\text{miles}\)

 

The time frame we are given for the closing of the gap is from 3 pm to 4:30 pm.

 

\(\text{T}=1.5\;\text{hours}=\frac{3}{2}\;\text{hours}\)

 

\(\text{S}_\text{G}=\frac{\text{D}}{\text{T}}\)

 

\(\text{S}_\text{G}=\frac{30\;\text{miles}}{\frac{3}{2}\;\text{hours}}=30\times\frac{2}{3}=20\;\text{miles per hour}\)

 

\(20\;\text{miles per hour}=\text{S}_\text{C}-50\;\text{miles per hour}\)

 

\(20\;\text{miles per hour}+50\;\text{miles per hour}=\text{S}_\text{C}=70\;\text{miles per hour}\)

 



data sufficiency question from Magoosh, then review the video explanation.[/*]
[/list]




Let us know how you did on these practice questions in the comments below. If you’re looking for more GMAT motion problems, try out one of Magoosh’s GMAT plans, which comes with practice tests, video lessons, and study schedules. Good luck!

The post How to Solve GMAT Motion Problems appeared first on Magoosh Blog — GMAT® Exam.
This Blog post was imported into the forum automatically. We hope you found it helpful. Please use the Kudos button if you did, or please PM/DM me if you found it disruptive and I will take care of it. -BB
Magoosh GMAT Instructor
Joined: 28 Dec 2011
Posts: 4452
Own Kudos [?]: 28605 [0]
Given Kudos: 130
How to Solve GMAT Motion Problems [#permalink]
Expert Reply
FROM Magoosh Blog: How to Solve GMAT Motion Problems


Word Problems make up a majority of the quantitative section of the GMAT (almost 60 percent). Of the word problems they’ll face, students tend to need the most help with GMAT motion problems. This type of problem centers around the “dust” formula, which is short for Distance equals Speed multiplied by Time, or \(\text{D}\times\text{S}=\text{T}\). But there are many varieties of motion problem, and we will discuss techniques for each of them. At the end of this article, you’ll also find motion word problems with solutions for you to test your knowledge!

The Distance Equation[/*]
[*]Multi-Segment Motion Problems[/*]
[*]Average Speed[/*]
[*]Multiple Traveler Questions[/*]
[*]Shrinking and Expanding Gaps[/*]
[*]Data Sufficiency[/*]
[*]GMAT Motion Problems Review: Practice Problems[/*]
[/list]
Distance equals Speed multiplied by Time, or \(\text{D}\times\text{S}=\text{T}\). If you learn this basic equation well, you’ll be able to dust your math troubles away! (Insert rimshot.)

 

We can rearrange this formula to determine that Speed is equal to Distance divided by Time, and that Time is equal to Distance divided by Speed. Distance is the measurement of how far apart objects, people, or points are. Speed is the rate at which someone or something is traveling. Time is how long it takes to travel.

 

Let’s demonstrate this. Walking at a constant rate of 160 meters per hour, Monroe can cross a bridge in 2 hours. What is the length of the bridge?

 

Here, the length of the bridge is the distance Monroe must cross. Using Distance equals Speed multiplied by Time, we get:

 

\((160\;\text{meters per hour})\times(2\;\text{hours})=320\;\text{meters}\)

 

Seems simple enough so far, right? Let’s check out a few more GMAT motion problems.





Speed is equal to Distance divided by Time. Let’s take that a step further and talk about average speed. Average speed is defined as total distance traveled divided by the total time period spent traveling. This means that if you have a trip with multiple segments, you’ll want to take the sum of the distances of each segment and divide that by the sum of the times of each segment. Average speed captures the constant speed needed to travel the total distance in the total time.

 

Let’s demonstrate this. Koki drove 16 miles in 10 minutes, and then drove an additional 6 miles in 5 minutes. What is Koki’s average speed for the entire trip in miles per hour?

Click here for the answer and explanation

Well, average speed is the total distance divided by the total time.

 

\(\text{D}_\text{Total}=16\;\text{miles}+6\;\text{miles}=22\;\text{miles}\)

 

\(\text{T}_\text{Total}=10\;\text{minutes}+5\;\text{minutes}=15\;\text{minutes}\)

 

\(\text{S}_\text{Average}=\frac{\text{D}_\text{Total}}{\text{T}_\text{Total}}\)

 

\(\text{S}_\text{Average}=\frac{22\;\text{miles}}{15\;\text{minutes}}\times\frac{60\;\text{minutes}}{1\;\text{hour}}=88\;\text{miles per hour}\)

 

That’s straightforward enough, but what if we are not given any distances or times? It is possible to solve an average speed problem, even if all you are given are the different speeds in each segment of the trip. You might then think that average speed would just be the average of all of the speeds, but that is not correct.

 

Let’s say that Nathaniel drove from Gwenville to Samton at an average speed of 24 miles per hour. He then drove the same route on the return trip back from Samton to Gwenville at an average speed of 36 miles per hour. If you were asked to find Nathaniel’s average speed, it would not just be 30 miles per hour (the average of 24 and 36).

Click here to work through this problem

To see this, let’s go back to our MVP dust formula. Since there are two legs of the trip, we will have two equations. D1, S1, T1; D2, S2, T2. Because Nathaniel’s trip is a round trip, we can assume that D1and D2 are the same, so we will set both of them equal to D.

 

\(\text{D}_1=\text{D}\\\text{S}_1=24\;\text{miles per hour}\\\text{T}_1=\frac{D}{24\;\text{miles per hour}}\)

 

\(\text{D}_2=\text{D}\\\text{S}_2=36\;\text{miles per hour}\\\text{T}_2=\frac{D}{36\;\text{miles per hour}}\)

 

\(\text{D}_\text{Total}=\text{2D}\)

 

\(\text{T}_\text{Total}=\frac{D}{24\;\text{miles per hour}}+\frac{D}{36\;\text{miles per hour}}\)

 

\(\text{S}_\text{Average}=\frac{\text{D}_\text{Total}}{\text{T}_\text{Total}}\)

 

\(\text{S}_\text{Average}=\frac{\text{2D}}{\frac{D}{24\;\text{miles per hour}}+\frac{D}{36\;\text{miles per hour}}}\)

 

We can factor a D out of this fraction.

 

\(\text{S}_\text{Average}=\frac{2}{\frac{1}{24}+\frac{1}{36}}\;\text{miles per hour}\)

 

We can find a common denominator between 24 and 36.

 

\(\text{S}_\text{Average}=\frac{2}{\frac{3}{72}+\frac{2}{72}}\;\text{miles per hour}\)

 

\(\text{S}_\text{Average}=\frac{2}{\frac{5}{72}}\;\text{miles per hour}=\frac{2}{1}\times\frac{72}{5}=\frac{144}{5}\;\text{miles per hour}=28.8\;\text{miles per hour}\)

 

In summary, whenever you want to find the average speed of a round trip, and you are given the two segment speeds, you can put it in the form \(\text{S}_\text{Average}=\frac{\text{2}}{\frac{1}{\text{S}_1}+\frac{1}{\text{S}_2}}\). You can also use this formula to find one of the segment speeds, given the other segment speed and the average speed.





diagram.

 

Let’s say that a car and truck are moving in the same direction on the same highway. The truck is moving at 50 miles an hour, and the car is traveling at a constant speed. At 3 pm, the car is 30 miles behind the truck and at 4:30 pm, the car overtakes and passes the truck. What is the speed of the car?

Click here for the answer and explanation

The car and truck are moving in the same direction, and the car is gaining on the truck. This means that the gap between the vehicles is shrinking and that the gap rate is the difference of the two vehicles’ respective speeds.

 

\(\text{S}_\text{G}=\text{S}_\text{C}-\text{S}_\text{T}\)

 

The distance of the gap is initially 30 miles.

 

\(\text{D}=30\;\text{miles}\)

 

The time frame we are given for the closing of the gap is from 3 pm to 4:30 pm.

 

\(\text{T}=1.5\;\text{hours}=\frac{3}{2}\;\text{hours}\)

 

\(\text{S}_\text{G}=\frac{\text{D}}{\text{T}}\)

 

\(\text{S}_\text{G}=\frac{30\;\text{miles}}{\frac{3}{2}\;\text{hours}}=30\times\frac{2}{3}=20\;\text{miles per hour}\)

 

\(20\;\text{miles per hour}=\text{S}_\text{C}-50\;\text{miles per hour}\)

 

\(20\;\text{miles per hour}+50\;\text{miles per hour}=\text{S}_\text{C}=70\;\text{miles per hour}\)

 



data sufficiency question from Magoosh, then review the video explanation.[/*]
[/list]




Let us know how you did on these practice questions in the comments below. If you’re looking for more GMAT motion problems, try out one of Magoosh’s GMAT plans, which comes with practice tests, video lessons, and study schedules. Good luck!

The post How to Solve GMAT Motion Problems appeared first on Magoosh Blog — GMAT® Exam.
This Blog post was imported into the forum automatically. We hope you found it helpful. Please use the Kudos button if you did, or please PM/DM me if you found it disruptive and I will take care of it. -BB
Magoosh GMAT Instructor
Joined: 28 Dec 2011
Posts: 4452
Own Kudos [?]: 28605 [0]
Given Kudos: 130
Everything You Need to Know about Getting an MBA [#permalink]
Expert Reply
FROM Magoosh Blog: Everything You Need to Know about Getting an MBA


An MBA, or a “Masters of Business Administration,” is a postgraduate degree achieved after an MBA candidate successfully goes through a business-focused program, usually lasting 1 to 2 years, and can be full-time or part-time. MBA programs are intended to better prepare you for business management skills and to make graduates more competitive in applying for leadership roles, or to crack into popular and high-paying industries, like tech, high-finance, and consulting. Check out our answers to frequently asked questions about the MBA below!

What Is an MBA?
[*]What Are the Benefits of an MBA?[*]What Do MBAs Cost?[*]How Hard Is an MBA?[*]Who Is an Ideal MBA Candidate?[*]How Do I Apply for MBA Programs?[*]When Should I Start Applying for an MBA?[/list]



full-time and part-time MBA as both went through the same academic rigors.

Online MBA Programs
This is the broadest category here, as “online MBAs” could very well be EMBAs, full-time or part-time. Generally, the feature is in the name: it’s online, rather than in-person, but there are some hybrids. Scholarships, as noted above, are more plentiful for full-time in-person programs. Your mileage here will vary depending on the university, as many programs have deep wells of experience in facilitating connections and learning online, but others are a little newer to the scene and adapting to the increasing trend to move online.

  • We recommend asking an alumnus or alumna about their experience, especially since universities are, frankly, suspiciously non-critical about any drawbacks that may arise from an online experience over an in-person one.
They could very well be a great option, but do your research, and try to aim for more recent information, as they have changed drastically since 2020 and social distancing forced them into the spotlight.

Joint Degree Programs
Best for those who have a more defined post-MBA career path, joint-degrees have an industry-focus or field-focus. For example, Stanford has joint degrees focusing on education and business, while Darden has MD/MBA programs for those who are studying to be trained doctors and who need business skills. Graduates tend to be younger (24 to 30) and prefer full-time studies, as this degree is meant to gain access into a certain field by combining studies and credentials.



data on top-10 programs is abundant (hovering around $150,000 on average), but it’s a little more diffuse with other programs. Statistics for schools with lower rankings are less reliable but show an average of $50,000. That does not mean necessarily that the MBA world is bifurcated, but rather that there is a fairly broad spectrum. Also, it does mean that location matters and you should do your research on programs first based on fit, and then do digging into the benefits.[/list]

[*]Network – For many, this is the indisputable top benefit of an MBA course, which should definitely be weighed when considering online programs. You have access to an alumni network regardless (formally, through the university, and informally through shared bonds that are more likely to get you to connect with alumni in your chosen field).[/list]

[*]Higher employment rate – This is very program-specific, but generally, there is a higher chance of being employed consistently with an MBA. What is less clear, however, is the data behind their figures, which we’ll discuss a bit later here.[/list]

[*]Access to higher-paying roles – The highest-paying roles after graduation tend to be (in descending order of average pay): consulting, financial services, and tech. What is also interesting is that those who get an MBA have the biggest salary increases in these industries (again, in order): consulting, food/beverage, investment management, and private equity, averaging $40,000 higher salary per year after graduation.[/list]
On their own websites, MBA programs also highlight tangential benefits like increased confidence, better personal financial management, time-management skills, and broader worldview, but the data they used to gather such insights tends to focus on the gains in the careers and skills of those who ultimately decided to get an MBA. In other words, for those that decided it was worth it, it ended up being worth it.

What is missing from their research are the trajectories of those who decided on an MBA alternative, but those data are inherently hard to come by – as we don’t have a reliable A/B test to understand what would have happened had they gone a different route. That means that MBA programs say MBAs are worth it, but you should definitely dig deeper to make sure they’re right for you.



average debt accumulated from an MBA course is near $100,000, and so, most students are in fact paying a steep price for the programs.

Given the hefty tuition costs, you’re probably wondering if the MBA is truly worth it. That’s a topic for another post, but let’s just focus on the financial aspect of it. There are various rules of thumb for deciding this, but one that is fairly reasonable is this: the bump in compensation you expect in your first three years as an MBA graduate should cover the out-the-door cost of the MBA.

For example, if you paid 160,000 for a full-time MBA, but your post-MBA salary went from 90,000 to 110,000, you very well might want to give it a long, hard look to see if it was worth the cost (160,000 – 20,000×3 > 0) – as you have a deficit of 100,000 and likely a high-interest student loan capping your appetite for career mobility in the meantime. Conversely, if your post-MBA salary was up from 90,000 to 150,000 with a 160,000 price tag, now you’re looking at a much stronger business case (160,000 – 60,000×3 < 0), as that salary is now covering more 20,000 more than the cost of the program itself.

Rough math? Yes. Hard to calculate future salary? Of course! The key point to take away here is to think of the MBA as any savvy business person would: what is the probable ROI (return on investment) and what is the opportunity cost? In other words, ask yourself: will it be worth its cost, and what else could I be doing at the same time? You’ll also be spending at least a year on the degree itself and likely 100+ hours on the application itself, so time is also a consideration.



and in 2020, which is when many professionals decide that they’d like to boost their credentials and skills during a time with more uncertainty. Acceptance rates for universities have gone down recently due to an influx of applicants, hovering around 5% for most top-10 programs, for example.





short-list of schools, the easiest way to apply is to fill out the application online on the specific program’s website. Luckily, it appears that, while the details can change, the portal in which you’ll put your information is very similar across most colleges. In addition to your undergraduate degree and transcripts from your degrees (likely going back to high school), here are additional admissions requirements and their considerations:

Two letters of reference: Please check the university itself for the format, but luckily this tends to be uniform: (1) describe your relationship to the candidate, and (2) how do they compare to their peers. You can probably tell that this has a distinct bias for managers. It may feel awkward to ask your current employer for a recommendation, so clients, former managers, and professors could also be options. You will then need to give the contact information for your recommenders, who will then be emailed a link to fill in a page with their letter and often to answer questions about your aptitude (on a score from 1-9, with 9 being the highest, for each indicator).

GMAT/GRE score (or an Executive Assessment for EMBAS): Check the requirements for your program, as some schools will even accept “expired” GMAT scores (older than 5 years).

TOEFL or IELTS score: If your undergrad was not taught in English, it’s likely that you’ll be required to provide a TOEFL or IELTS score. This is program-specific, and some have loosened their requirement to look at other representations of English ability.

Admissions essays: The typical essays are “Why ____ program” and “Tell us about you,” but the recent trend has been for universities has been to ask more unique questions (see Duke’s “25 random things about yourself” essay, for example). There is also an “essay” asking for any additional information, which is an opportunity to talk about mitigating factors and other considerations but is not typically written in an essay-like way as the others. Most programs expect a simple bullet-point list for the “Anything else” essay.

An application fee: Fees can range from $50 to $300 but the idea is that each application is given consideration and so this covers the university’s overhead. Be sure to look up the university’s “fee waiver” for which many program applicants can qualify.

And the last thing you’ll need? Patience. For larger programs, it can take two or more months to hear back about interviews, but luckily their timelines are transparently communicated on their websites. It’s well worth it to first learn about programs, and then to mark the major dates in your calendar.



need more time to increase your GMAT score. For the overwhelming majority of programs that begin in the Fall, your rough timeline is:

Round 1 Deadlines: June to September of the preceding year (for example, a 2022 to 2023 program will close applications in Summer 2021). Start preparing your application in February.

Round 2 Deadlines: December to February before the program (for example, a 2022 to 2023 program will close applications in the preceding Winter). Start preparing your application by September.

Round 3 Deadlines: March to May of the same year (for example, a 2022 to 2023 program will close applications in Summer 2022). Start preparing your application in December.

While we’re on the subject, you might be wondering in which round you should apply. That’s a tough question, but overall the rule is “the earlier, the better,” as Round 2 tends to have more applicants than Round 1 and more in Round 3 than 2. The tough situation in which many applicants find themselves is weighing between Round 3 of this year or Round 1 of next year. For that, we don’t have an easy answer, but the question you should ask yourself is: will my application be substantially better in three months?



What You Need to Know about MBAs: A Summary
An MBA can be valuable for your career, but it will likely come with a considerable price tag and 1-2 years of study (on top of around 3 months of preparation), so it’s well worth doing research and some serious introspection with the question: is this right for me? Also keep in mind that not all MBA programs are made the same, as there are degrees that may very well be more tailored to your career interests, or perhaps, frankly, none at all!

If you’ve made it this far, congratulate yourself for doing research into what is right for you, and we wish you luck on whatever next steps you take. Let us know in the comments, are you planning on getting an MBA?

The post Everything You Need to Know about Getting an MBA appeared first on Magoosh Blog — GMAT® Exam.
This Blog post was imported into the forum automatically. We hope you found it helpful. Please use the Kudos button if you did, or please PM/DM me if you found it disruptive and I will take care of it. -BB
Magoosh GMAT Instructor
Joined: 28 Dec 2011
Posts: 4452
Own Kudos [?]: 28605 [0]
Given Kudos: 130
How to Solve GMAT Motion Problems [#permalink]
Expert Reply
FROM Magoosh Blog: How to Solve GMAT Motion Problems


Word Problems make up a majority of the quantitative section of the GMAT (almost 60 percent). Of the word problems they’ll face, students tend to need the most help with GMAT motion problems. This type of problem centers around the “dust” formula, which is short for Distance equals Speed multiplied by Time, or \(text{D}timestext{S}=text{T}\). But there are many varieties of motion problem, and we will discuss techniques for each of them. At the end of this article, you’ll also find motion word problems with solutions for you to test your knowledge!

The Distance Equation[/*]
[*]Multi-Segment Motion Problems[/*]
[*]Average Speed[/*]
[*]Multiple Traveler Questions[/*]
[*]Shrinking and Expanding Gaps[/*]
[*]Data Sufficiency[/*]
[*]GMAT Motion Problems Review: Practice Problems[/*]
[/list]
Distance equals Speed multiplied by Time, or \(text{D}timestext{S}=text{T}\). If you learn this basic equation well, you’ll be able to dust your math troubles away! (Insert rimshot.)



 

We can rearrange this formula to determine that Speed is equal to Distance divided by Time.



And Time is equal to Distance divided by Speed.



  • Distance is the measurement of how far apart objects, people, or points are.
  • Speed is the rate at which someone or something is traveling.
  • Time is how long it takes to travel.
 

Let’s demonstrate this. Walking at a constant rate of 160 meters per hour, Monroe can cross a bridge in 2 hours. What is the length of the bridge?

 

Here, the length of the bridge is the distance Monroe must cross. Using Distance equals Speed multiplied by Time, we get:

 

\((160;text{meters per hour})times(2;text{hours})=320;text{meters}\)

 

Seems simple enough so far, right? Let’s check out a few more GMAT motion problems.





Speed is equal to Distance divided by Time. Let’s take that a step further and talk about average speed. Average speed is defined as total distance traveled divided by the total time period spent traveling. This means that if you have a trip with multiple segments, you’ll want to take the sum of the distances of each segment and divide that by the sum of the times of each segment. Average speed captures the constant speed needed to travel the total distance in the total time.

 

Let’s demonstrate this. Koki drove 16 miles in 10 minutes, and then drove an additional 6 miles in 5 minutes. What is Koki’s average speed for the entire trip in miles per hour?

Click here for the answer and explanation

Well, average speed is the total distance divided by the total time.

 

\(text{D}_text{Total}=16;text{miles}+6;text{miles}=22;text{miles}\)

 

\(text{T}_text{Total}=10;text{minutes}+5;text{minutes}=15;text{minutes}\)

 

\(text{S}_text{Average}=frac{text{D}_text{Total}}{text{T}_text{Total}}\)

 

\(text{S}_text{Average}=frac{22;text{miles}}{15;text{minutes}}timesfrac{60;text{minutes}}{1;text{hour}}=88;text{miles per hour}\)

 

That’s straightforward enough, but what if we are not given any distances or times? It is possible to solve an average speed problem, even if all you are given are the different speeds in each segment of the trip. You might then think that average speed would just be the average of all of the speeds, but that is not correct.

 

Let’s say that Nathaniel drove from Gwenville to Samton at an average speed of 24 miles per hour. He then drove the same route on the return trip back from Samton to Gwenville at an average speed of 36 miles per hour. If you were asked to find Nathaniel’s average speed, it would not just be 30 miles per hour (the average of 24 and 36).

Click here to work through this problem

To see this, let’s go back to our MVP dust formula. Since there are two legs of the trip, we will have two equations. D1, S1, T1; D2, S2, T2. Because Nathaniel’s trip is a round trip, we can assume that D1and D2 are the same, so we will set both of them equal to D.

 

\(text{D}_1=text{D}\text{S}_1=24;text{miles per hour}\text{T}_1=frac{D}{24;text{miles per hour}}\)

 

\(text{D}_2=text{D}\text{S}_2=36;text{miles per hour}\text{T}_2=frac{D}{36;text{miles per hour}}\)

 

\(text{D}_text{Total}=text{2D}\)

 

\(text{T}_text{Total}=frac{D}{24;text{miles per hour}}+frac{D}{36;text{miles per hour}}\)

 

\(text{S}_text{Average}=frac{text{D}_text{Total}}{text{T}_text{Total}}\)

 

\(text{S}_text{Average}=frac{text{2D}}{frac{D}{24;text{miles per hour}}+frac{D}{36;text{miles per hour}}}\)

 

We can factor a D out of this fraction.

 

\(text{S}_text{Average}=frac{2}{frac{1}{24}+frac{1}{36}};text{miles per hour}\)

 

We can find a common denominator between 24 and 36.

 

\(text{S}_text{Average}=frac{2}{frac{3}{72}+frac{2}{72}};text{miles per hour}\)

 

\(text{S}_text{Average}=frac{2}{frac{5}{72}};text{miles per hour}=frac{2}{1}timesfrac{72}{5}=frac{144}{5};text{miles per hour}=28.8;text{miles per hour}\)

 

In summary, whenever you want to find the average speed of a round trip, and you are given the two segment speeds, you can put it in the form \(text{S}_text{Average}=frac{text{2}}{frac{1}{text{S}_1}+frac{1}{text{S}_2}}\). You can also use this formula to find one of the segment speeds, given the other segment speed and the average speed.





diagram.

 

Let’s say that a car and truck are moving in the same direction on the same highway. The truck is moving at 50 miles an hour, and the car is traveling at a constant speed. At 3 pm, the car is 30 miles behind the truck and at 4:30 pm, the car overtakes and passes the truck. What is the speed of the car?

Click here for the answer and explanation

The car and truck are moving in the same direction, and the car is gaining on the truck. This means that the gap between the vehicles is shrinking and that the gap rate is the difference of the two vehicles’ respective speeds.

 

\(text{S}_text{G}=text{S}_text{C}-text{S}_text{T}\)

 

The distance of the gap is initially 30 miles.

 

\(text{D}=30;text{miles}\)

 

The time frame we are given for the closing of the gap is from 3 pm to 4:30 pm.

 

\(text{T}=1.5;text{hours}=frac{3}{2};text{hours}\)

 

\(text{S}_text{G}=frac{text{D}}{text{T}}\)

 

\(text{S}_text{G}=frac{30;text{miles}}{frac{3}{2};text{hours}}=30timesfrac{2}{3}=20;text{miles per hour}\)

 

\(20;text{miles per hour}=text{S}_text{C}-50;text{miles per hour}\)

 

\(20;text{miles per hour}+50;text{miles per hour}=text{S}_text{C}=70;text{miles per hour}\)

 



data sufficiency question from Magoosh, then review the video explanation.[/list]




Let us know how you did on these practice questions in the comments below. If you’re looking for more GMAT motion problems, try out one of Magoosh’s GMAT plans, which comes with practice tests, video lessons, and study schedules. Good luck!

The post How to Solve GMAT Motion Problems appeared first on Magoosh Blog — GMAT® Exam.
This Blog post was imported into the forum automatically. We hope you found it helpful. Please use the Kudos button if you did, or please PM/DM me if you found it disruptive and I will take care of it. -BB
Magoosh GMAT Instructor
Joined: 28 Dec 2011
Posts: 4452
Own Kudos [?]: 28605 [0]
Given Kudos: 130
Everything You Need to Know about Getting an MBA [#permalink]
Expert Reply
FROM Magoosh Blog: Everything You Need to Know about Getting an MBA


An MBA, or a “Masters of Business Administration,” is a postgraduate degree achieved after an MBA candidate successfully goes through a business-focused program, usually lasting 1 to 2 years, and can be full-time or part-time. MBA programs are intended to better prepare you for business management skills and to make graduates more competitive in applying for leadership roles, or to crack into popular and high-paying industries, like tech, high-finance, and consulting. Check out our answers to frequently asked questions about the MBA below!

What Is an MBA?
[*]What Are the Benefits of an MBA?[*]What Do MBAs Cost?[*]How Hard Is an MBA?[*]Who Is an Ideal MBA Candidate?[*]How Do I Apply for MBA Programs?[*]When Should I Start Applying for an MBA?[/list]



full-time and part-time MBA as both went through the same academic rigors.

Online MBA Programs
This is the broadest category here, as “online MBAs” could very well be EMBAs, full-time or part-time. Generally, the feature is in the name: it’s online, rather than in-person, but there are some hybrids. Scholarships, as noted above, are more plentiful for full-time in-person programs. Your mileage here will vary depending on the university, as many programs have deep wells of experience in facilitating connections and learning online, but others are a little newer to the scene and adapting to the increasing trend to move online.

  • We recommend asking an alumnus or alumna about their experience, especially since universities are, frankly, suspiciously non-critical about any drawbacks that may arise from an online experience over an in-person one.
They could very well be a great option, but do your research, and try to aim for more recent information, as they have changed drastically since 2020 and social distancing forced them into the spotlight.

Joint Degree Programs
Best for those who have a more defined post-MBA career path, joint-degrees have an industry-focus or field-focus. For example, Stanford has joint degrees focusing on education and business, while Darden has MD/MBA programs for those who are studying to be trained doctors and who need business skills. Graduates tend to be younger (24 to 30) and prefer full-time studies, as this degree is meant to gain access into a certain field by combining studies and credentials.



data on top-10 programs is abundant (hovering around $150,000 on average), but it’s a little more diffuse with other programs. Statistics for schools with lower rankings are less reliable but show an average of $50,000. That does not mean necessarily that the MBA world is bifurcated, but rather that there is a fairly broad spectrum. Also, it does mean that location matters and you should do your research on programs first based on fit, and then do digging into the benefits.[/list]

[*]Network – For many, this is the indisputable top benefit of an MBA course, which should definitely be weighed when considering online programs. You have access to an alumni network regardless (formally, through the university, and informally through shared bonds that are more likely to get you to connect with alumni in your chosen field).[/list]

[*]Higher employment rate – This is very program-specific, but generally, there is a higher chance of being employed consistently with an MBA. What is less clear, however, is the data behind their figures, which we’ll discuss a bit later here.[/list]

[*]Access to higher-paying roles – The highest-paying roles after graduation tend to be (in descending order of average pay): consulting, financial services, and tech. What is also interesting is that those who get an MBA have the biggest salary increases in these industries (again, in order): consulting, food/beverage, investment management, and private equity, averaging $40,000 higher salary per year after graduation.[/list]
On their own websites, MBA programs also highlight tangential benefits like increased confidence, better personal financial management, time-management skills, and broader worldview, but the data they used to gather such insights tends to focus on the gains in the careers and skills of those who ultimately decided to get an MBA. In other words, for those that decided it was worth it, it ended up being worth it.

What is missing from their research are the trajectories of those who decided on an MBA alternative, but those data are inherently hard to come by – as we don’t have a reliable A/B test to understand what would have happened had they gone a different route. That means that MBA programs say MBAs are worth it, but you should definitely dig deeper to make sure they’re right for you.



average debt accumulated from an MBA course is near $100,000, and so, most students are in fact paying a steep price for the programs.

Given the hefty tuition costs, you’re probably wondering if the MBA is truly worth it. That’s a topic for another post, but let’s just focus on the financial aspect of it. There are various rules of thumb for deciding this, but one that is fairly reasonable is this: the bump in compensation you expect in your first three years as an MBA graduate should cover the out-the-door cost of the MBA.

For example, if you paid 160,000 for a full-time MBA, but your post-MBA salary went from 90,000 to 110,000, you very well might want to give it a long, hard look to see if it was worth the cost (160,000 – 20,000×3 > 0) – as you have a deficit of 100,000 and likely a high-interest student loan capping your appetite for career mobility in the meantime. Conversely, if your post-MBA salary was up from 90,000 to 150,000 with a 160,000 price tag, now you’re looking at a much stronger business case (160,000 – 60,000×3 < 0), as that salary is now covering more 20,000 more than the cost of the program itself.

Rough math? Yes. Hard to calculate future salary? Of course! The key point to take away here is to think of the MBA as any savvy business person would: what is the probable ROI (return on investment) and what is the opportunity cost? In other words, ask yourself: will it be worth its cost, and what else could I be doing at the same time? You’ll also be spending at least a year on the degree itself and likely 100+ hours on the application itself, so time is also a consideration.



and in 2020, which is when many professionals decide that they’d like to boost their credentials and skills during a time with more uncertainty. Acceptance rates for universities have gone down recently due to an influx of applicants, hovering around 5% for most top-10 programs, for example.





short-list of schools, the easiest way to apply is to fill out the application online on the specific program’s website. Luckily, it appears that, while the details can change, the portal in which you’ll put your information is very similar across most colleges. In addition to your undergraduate degree and transcripts from your degrees (likely going back to high school), here are additional admissions requirements and their considerations:

Two letters of reference: Please check the university itself for the format, but luckily this tends to be uniform: (1) describe your relationship to the candidate, and (2) how do they compare to their peers. You can probably tell that this has a distinct bias for managers. It may feel awkward to ask your current employer for a recommendation, so clients, former managers, and professors could also be options. You will then need to give the contact information for your recommenders, who will then be emailed a link to fill in a page with their letter and often to answer questions about your aptitude (on a score from 1-9, with 9 being the highest, for each indicator).

GMAT/GRE score (or an Executive Assessment for EMBAS): Check the requirements for your program, as some schools will even accept “expired” GMAT scores (older than 5 years).

TOEFL or IELTS score: If your undergrad was not taught in English, it’s likely that you’ll be required to provide a TOEFL or IELTS score. This is program-specific, and some have loosened their requirement to look at other representations of English ability.

Admissions essays: The typical essays are “Why ____ program” and “Tell us about you,” but the recent trend has been for universities has been to ask more unique questions (see Duke’s “25 random things about yourself” essay, for example). There is also an “essay” asking for any additional information, which is an opportunity to talk about mitigating factors and other considerations but is not typically written in an essay-like way as the others. Most programs expect a simple bullet-point list for the “Anything else” essay.

An application fee: Fees can range from $50 to $300 but the idea is that each application is given consideration and so this covers the university’s overhead. Be sure to look up the university’s “fee waiver” for which many program applicants can qualify.

And the last thing you’ll need? Patience. For larger programs, it can take two or more months to hear back about interviews, but luckily their timelines are transparently communicated on their websites. It’s well worth it to first learn about programs, and then to mark the major dates in your calendar.



need more time to increase your GMAT score. For the overwhelming majority of programs that begin in the Fall, your rough timeline is:

Round 1 Deadlines: June to September of the preceding year (for example, a 2022 to 2023 program will close applications in Summer 2021). Start preparing your application in February.

Round 2 Deadlines: December to February before the program (for example, a 2022 to 2023 program will close applications in the preceding Winter). Start preparing your application by September.

Round 3 Deadlines: March to May of the same year (for example, a 2022 to 2023 program will close applications in Summer 2022). Start preparing your application in December.

While we’re on the subject, you might be wondering in which round you should apply. That’s a tough question, but overall the rule is “the earlier, the better,” as Round 2 tends to have more applicants than Round 1 and more in Round 3 than 2. The tough situation in which many applicants find themselves is weighing between Round 3 of this year or Round 1 of next year. For that, we don’t have an easy answer, but the question you should ask yourself is: will my application be substantially better in three months?



What You Need to Know about MBAs: A Summary
An MBA can be valuable for your career, but it will likely come with a considerable price tag and 1-2 years of study (on top of around 3 months of preparation), so it’s well worth doing research and some serious introspection with the question: is this right for me? Also keep in mind that not all MBA programs are made the same, as there are degrees that may very well be more tailored to your career interests, or perhaps, frankly, none at all!

If you’ve made it this far, congratulate yourself for doing research into what is right for you, and we wish you luck on whatever next steps you take. Let us know in the comments, are you planning on getting an MBA?

The post Everything You Need to Know about Getting an MBA appeared first on Magoosh Blog — GMAT® Exam.
This Blog post was imported into the forum automatically. We hope you found it helpful. Please use the Kudos button if you did, or please PM/DM me if you found it disruptive and I will take care of it. -BB
Magoosh GMAT Instructor
Joined: 28 Dec 2011
Posts: 4452
Own Kudos [?]: 28605 [0]
Given Kudos: 130
Is an MBA Worth It? [#permalink]
Expert Reply
FROM Magoosh Blog: Is an MBA Worth It?


Is an MBA worth it? Right now, most traditional MBAs aren’t. But some MBAs still are, depending on your situation. With that in mind, there are a lot of factors to consider before you completely write off doing a Master of Business Administration.

MBAs are traditionally positioned as degrees that better prepare you for business management skills and that make graduates more competitive in applying for leadership roles, or to crack into popular and high-paying industries, like tech, high-finance, and consulting.

However, there’s a recent declining interest in MBAs, even as applicant numbers are rising with economic stress. The main concern is that they aren’t able to keep pace with technological innovations that are changing the way businesses run. Beyond that, costs for traditional programs can be high—we’re talking hundreds of thousands of dollars.

So, with all that in mind, the real question we should be asking is…which MBA programs are worth it?

Which MBA Programs Are Worth it?
Before you can know whether an MBA is worth it, you need to ask: what does an MBA get you? As we detail extensively in our general post about MBAs, here are the commonly cited benefits:

  • Coursework and skills development
  • Higher income potential/earning power
  • Networking opportunities
  • Higher employment rate
  • Access to higher-paying roles
But the fact is that not all of these benefits are still true, particularly during the COVID-19 pandemic. If you choose a program that’s not right for you, you may end up on the losing side of the risk/reward equation. After all, not all MBA programs are made the same, as there are degrees that may very well be more tailored to your career interests, or perhaps, frankly, none at all!

Here’s the rundown on the pros and cons of different MBA programs right now, as well as who they might be worth it for.

The Value of a Full-Time MBA
A full-time MBA generally is crafted for those with 2+ years of post-bachelor’s work experience. These are the traditional MBAs that most people picture, involving coursework guided by a core curriculum, team capstones, and an MBA thesis supported by faculty. Full-time MBA programs are typically two years, but sometimes the two-year curriculum is squeezed into one-year programs, making for a much more intense experience.

In 2021, a full-time MBA may not have the same weight as it did even two years ago. We’re not living in normal times. You won’t be able to count on a lot of the benefits of a traditional full-time MBA program for your career, given the economic uncertainty that’s accompanying the pandemic and the resulting rise in applicants. It’s hard to say what the hiring process will look like for professionals a year from now.

Also, given that most traditional MBA programs aren’t meeting in-person due to the COVID-19 pandemic, the huge price tag that comes attached means that you’re getting a lot less value than you normally would. No in-person classes, far less networking, scarce internship opportunities…

There are a few exceptions, of course. Two big factors to take into consideration are rankings and scholarships A top business school degree (think: Harvard, Wharton) will almost always be an asset, coming with huge opportunities and higher earning potential, even in a terrible job market. And if you can offset the price with scholarships, the math just might add up.

That’s not snobbery—it’s based on data. And the data on top-10 programs is abundant (average MBA salaries hovering around $173,860 on average), but it’s a little more diffuse with other programs. When looking at all MBA programs, the average falls sharply to $106,757. And keep in mind that’s the average across all programs; it’s weighted by those higher salaries from top-10 graduates.

The Value of a Joint-degree MBA
Joint-degree programs typically combine the traditional MBA with a traditional program in another field, such as an MD or Master in Public Policy, to allow candidates to build expertise at the intersection of business and the other field in question. Joint-degrees typically last the same amount of time minus one year of the combined length of the programs. Because joint-degrees are more expensive than the traditional MBA alone, the cost-benefit analysis is arguably more critical, especially in these times.

In general, joint-degree programs are best for those who have a more defined post-MBA career path. It may also be worth it if you can find a prestigious fellowship (ahem) or scholarship to offset the cost.

The Value of a Part-Time MBA
Although part-time MBAs lack many of the networking benefits of a full-time MBA and it’s hard to qualify for a grant or scholarship for these programs, they do provide access to some alumni networks and on paper (i.e. your CV), there is virtually no difference between a part-time and full-time MBA. A part-time MBA is worth it for those who need the degree itself for a particular purpose, such as getting a promotion in their current company, and can take on the financial burden without going into significant debt.

The Value of Online MBA Programs
Although there could be an argument for ALL MBA programs qualifying as online MBA programs these days, true online MBA programs are meant to be conducted in an online setting. Whether an online MBA is worth it depends entirely on the program itself in terms of the education and opportunities it offers you. This option is most valuable to students who need flexibility and are assured of their program’s quality and offerings. If you’re considering getting an online MBA, make sure to connect with alumni, as rankings can only tell you so much in the new landscape.

Is an MBA Worth It? A Summary
Any MBA program is a big investment. Here’s when they’re worth it:

  • Traditional MBA: You get into one of the top schools and scholarships absorb some of the cost
  • Joint MBA program: You have a particular career goal in mind that requires both degrees
  • Part-time MBA: You need the degree for a specific reason that does not rely on networking or recruiting opportunities
  • Online MBA: You need a flexible option and can verify that the program is well-adapted to online platform
Of course, earnings potential isn’t the only benefit of an MBA. On their own websites, MBA programs also highlight tangential benefits like increased confidence, better personal financial management, time-management skills, and broader worldview, but the data they used to gather such insights tends to focus on the gains in the careers and skills of those who ultimately decided to get an MBA. In other words, for those that decided it was worth it, it ended up being worth it.

What is missing from their research are the trajectories of those who decided on an MBA alternative, but those data are inherently hard to come by – as we don’t have a reliable A/B test to understand what would have happened had they gone a different route. That means that MBA programs say MBAs are worth it, but you should definitely dig deeper to make sure they’re right for you.

The post Is an MBA Worth It? appeared first on Magoosh Blog — GMAT® Exam.
This Blog post was imported into the forum automatically. We hope you found it helpful. Please use the Kudos button if you did, or please PM/DM me if you found it disruptive and I will take care of it. -BB
Magoosh GMAT Instructor
Joined: 28 Dec 2011
Posts: 4452
Own Kudos [?]: 28605 [0]
Given Kudos: 130
GMAT Fee Waiver: How to Apply [#permalink]
Expert Reply
FROM Magoosh Blog: GMAT Fee Waiver: How to Apply


The GMAT can be a pricey test. In the United States, it costs $275 for the exam alone (though it’s less in other locations, like the $250 India GMAT fee). If this isn’t in your budget, you do have options—primarily getting a GMAT fee waiver. We’ll take a look at what GMAT fee waivers are and what they cover, how to get one, and how to use them.

What are GMAT fee waivers?[/*]
[*]How do I get a fee waiver for GMAT?[/*]
[*]Which business schools accept GMAT fee waivers?[/*]
[*]How do I use my fee waiver?[/*]
[/list]
GMAT exam cost. In other words, they let you take the exam for a reduced price or even for free.

Because students apply to their schools, not the GMAC (the Graduate Management Admission Council, AKA the the test-maker) for these waivers, it’s your school that ultimately decides how much of the fee gets waived.

As you look for ways to reduce GMAT costs, keep in mind that a GMAT fee waiver is different from other types of waivers!

  • A GMAT waiver, for example, is something that B-school admissions committees use. This type of waiver lets the program admit you even if you’ve skipped the test altogether. Almost always, adcoms have policies about this type of waiver, based on professional experience (work experience), educational history, or other criteria.
  • Similarly, a waived application fee also comes from the schools you’re applying to. Each MBA program will have its own criteria for application fee waivers, so contact them directly.
  • Finally, a waived rescheduling fee is a fee you don’t have to pay when you change your GMAT test date. The GMAC has waived some of these fees during the COVID-19 pandemic and may consider others on a case-by-case basis.
 



 

DANTES (Defense Activity for Non-Traditional Education Support) reimbursement.

This only includes the test fee itself. It won’t pay off late fees, rescheduling fees, or other associated costs.

You’ll also have to pay the money up-front yourself and then apply for DANTES reimbursement after you take the test. You need both your receipt and your results to get paid back. Make sure to apply within 90 days of your test date by submitting an online request.

For military veterans, the GI Bill will reimburse you for the cost of one test.

Note that GRE waivers are also available, which some MBA programs will accept.

GMAT Fee Waivers and Covid-19
While lots of business schools have adjusted to the COVID-19 pandemic, one part remains the same: the fee waiver program. This hasn’t been expanded or changed as of right now.

However, the good news is that 38% of MBA programs responding to the GMAC’s survey have reduced or removed application fees, so you may still end up saving money during the application process.

 



 

University of Arkansas at Little Rock[*]Eastern Illinois University[*]University of North Dakota[*]University of Rhode Island[*]SUNY Polytechnic Institute[*]Cal State LA[/list]
 



 

GMAT Fee Waiver: How to Apply appeared first on Magoosh Blog — GMAT® Exam.
This Blog post was imported into the forum automatically. We hope you found it helpful. Please use the Kudos button if you did, or please PM/DM me if you found it disruptive and I will take care of it. -BB
Magoosh GMAT Instructor
Joined: 28 Dec 2011
Posts: 4452
Own Kudos [?]: 28605 [0]
Given Kudos: 130
GMAT Registration | How to Register for the GMAT [#permalink]
Expert Reply
FROM Magoosh Blog: GMAT Registration | How to Register for the GMAT


You’ve decided to take the next step in your career and go to business school. Congratulations! Now, down to brass tacks: GMAT registration. From ID to fees, here’s everything you need to know about how to register for the GMAT!

How do I register for the GMAT?[/*]
[*]How much is the GMAT registration fee?[/*]
[*]How many days before should I register for the GMAT?[/*]
[*]What happens after I register for the GMAT?[/*]
[/list]
GMAC’s “Create Your Account” page to get started. Once that’s done, you’ll have the ability to browse and pick your GMAT location and see your options for GMAT dates.[/*]
[*]If you’re requesting GMAT accommodations, follow this process—you’ll need to get approval before registering. For any questions, email testingaccommodations@gmac.com or call customer service toll-free +1-800-717-GMAT.[/*]
[/list]
[/*]
[*]Next, find a GMAT test center or prepare for the at-home online GMAT. (For GMAT registrations in early 2021, check here for the latest information). To find a GMAT location near you, use the GMAT’s official test center search page. And remember, the farther in advance you book your GMAT, the more options you’ll have for both GMAT locations and GMAT dates.[/*]
[*]Once you are registered to take the GMAT and know your GMAT test center location, you’re ready to actually select your GMAT test date. Note that you have to pick a date and time to complete your GMAT test registration. Log in to your GMAT account and follow the on-screen instructions.[/*]
[/list]
GMAT Registration Checklist
For GMAT exam registration, you’ll need to give the following info on the GMAT appointment scheduling form:

  • Full name (this needs to match the name on your ID exactly, so have your passport handy!
  • Email
  • Mailing address
  • Phone
  • Gender
  • Date of birth
  • Nationality
  • Credit card or debit card (VISA, Mastercard, American Express, or Discover)
If you don’t have or don’t want to use a credit/debit card, you can also pay the fee by money order or personal check; follow the instructions here.

You’ll also have the option of entering your education history, your employment history, and your native language, but these aren’t mandatory parts of GMAT registration.

How many times can I sign up for the exam?
You can only be registered for one GMAT administration at a time. In other words, even if you’re planning on retaking the exam, you’ll need to schedule your second GMAT registration after you take the first test. Yes, that means you’ll need to pay the GMAT exam registration fees again, too.

There’s also a limit to how many times you can take the GMAT in a single calendar year: 5 times. Once you’ve reached your GMAT registration attempt limit, you will have to wait for next year.

Finally, keep in mind that you’ll need to have a 16-day gap between official GMAT exams.

 



 

India, Africa, the Middle East, Asia Pacific and most of the world is $250 (USD), but that price can differ in the USA, Canada, and Europe—see our article “How Much Does the GMAT Cost?” for details.

 



 



 

GMAT handbook from the Graduate Management Admission Council to get your bearings. If you haven’t already started studying, or if you need help organizing your schedule, Magoosh’s GMAT study plans can help get you where you need to be. Or dive right into a GMAT practice test to see how your current skills measure up and where you need to improve before your test day!

The post GMAT Registration | How to Register for the GMAT appeared first on Magoosh Blog — GMAT® Exam.
This Blog post was imported into the forum automatically. We hope you found it helpful. Please use the Kudos button if you did, or please PM/DM me if you found it disruptive and I will take care of it. -BB
Magoosh GMAT Instructor
Joined: 28 Dec 2011
Posts: 4452
Own Kudos [?]: 28605 [0]
Given Kudos: 130
How to Tackle Critical Reading Assumption Questions [#permalink]
Expert Reply
FROM Magoosh Blog: How to Tackle Critical Reading Assumption Questions


Assumption questions ask you to find the unstated link between a question’s premise and its conclusion. Assumptions are crucial in understanding and refuting arguments, so they play a large role in two major Critical Reasoning question types. In this post, we’ll cover GMAT Critical Reasoning tips and practice questions to help you tackle assumption questions.

GMAT Critical Reasoning Tips: How to Tackle Assumption Questions
Luckily, arguments on GMAT Critical Reasoning questions are relatively formulaic, so let’s go over the basics first:

  • A premise is the starting point of the argument.
  • The conclusion is what the author wants you to believe by the end of the argument.
  • The assumption is the missing link between the premise and conclusion. Think of it like the linchpin holding the whole thing together. You can strengthen an argument by validating its assumption, or weaken the argument by denying the assumption.
Assumption questions will usually ask you, “Which would most strengthen the argument?” or “Which of the following would most weaken the argument?” (the latter is one of the most common on Critical Reasoning).

Make Your Assumption a General Statement
This is a crucial point to remember: assumptions are most often general statements, not specific statements. When you identify the assumption, you can omit any specific people, places, or items mentioned.

If my premise is “Fred has quality A,” and my conclusion is “Therefore, Fred has quality B,” Fred is a specific person that we can omit (sorry, Fred). The assumption would be something like “most/all folks who have quality A also have quality B.”

Identify the Assumption
Isolating an assumption is an important skill and one of our favorite GMAT Critical Reasoning tips. Let’s try it with this argument:

Hawaii is a place with beautiful scenery. Therefore, people there must have trouble concentrating for any length of time at all.

  • The premise is “Hawaii is a place with beautiful scenery.” (We can safely assume that at least 99 out of a hundred people would agree with that!) Hawaii is the specific, so you can omit that—the final premise has to do with a “place with beautiful scenery.”
  • The conclusion is “trouble concentrating.”
  • The assumption must provide a link. If we put those together with a strong logical connection, we get this assumption: “People in places with beautiful scenery generally have trouble concentrating.” Even though it’s a little absurd, that’s a possible way to state the assumption!
It would most strengthen this argument if one could somehow provide data or evidence supporting this assumption. This argument would be weakened if we could cite data or evidence that directly contradicts the assumption.

Now, consider an argument you’re more likely to see on the GMAT:

Of all the companies in the steel industry in the last six months, only Amalgamated Ferric Industry (AFI) has tripled their advertising expenditures. No other steel company has increased advertising nearly that much. Therefore, in the coming months, we should see AFI gaining new customers at a rate that outpaces all its competitors.

  • If we drop the specifics, the premise is about increasing spending on advertising, and the conclusion is: more new customers. An assumption would link these.
  • A very broad assumption: “Companies that increase what they spend on advertising generally see an increase in new customers.”
  • A slightly more specific assumption: “When companies in the steel industry increase advertising, this generally results in more new customers.”
This is a relatively poor argument, and if we were asked for a statement to weaken it, the best choice would be something that zeroed in on the assumption. For example, something like Studies of companies in the steel industry show little correlation between advertising dollars and new customers strikes right at the center of the argument.

Use the Negation Test to Verify the Assumption
If you want to verify that your assumption is really the correct one, you can use the Negation Test—put simply, try negating the statement and seeing if the conclusion is still true. If you haven’t tried the Negation Test yet (another of our key GMAT Critical Reasoning tips!), then I would definitely recommend checking out our post and studying this powerful technique for isolating assumptions of arguments.

Practice Questions and Explanations

Prof. Hernandez’s monumental work The History of Central America covers everything about the region from the origin of the Mesoamerican period to the end of the Cold War. While the book has several informative maps and charts, many of the chapters spend less time describing facts and more time explaining Prof. Hernandez’s theories. Indeed, the last two chapters consist exclusively of his exposition of theory of the role of Central America in post WWII world politics. Therefore, properly speaking, this book is not a history book.

1. Which of the following is an assumption that supports drawing the conclusion above from the reasons given for that conclusion?

(A) Some history books consist almost exclusively of catalogues of historical facts.

(B) Different historians have different understanding of the relative importance between facts and theories within the study of history.

(C) Historians should be more explicit than most are now about the theoretical framework with which they write.

(D) History as a discipline is concerned only with historical facts, not with the theoretical explanations of those facts.

(E) Most books that present a wealth of historical facts include maps and charts as well.

Click here for the answer and video explanation!

In the twentieth century, the visual arts have embarked on major experimentation, from cubism to expressionism. While tastes always vary, there are certainly some people who find beautiful objects of each of the art movements of the first half of the twentieth century. In the latter half of the twentieth century, though, most works are so abstract or shocking that neither the critic nor the general public uses the word “beautiful” to describe them: indeed, sometimes late twentieth-century artists have, as one of their expressed goals, the creation of a work that no one could find beautiful. Whatever these artists are creating may be intellectually engaging at some level, but it is no longer art.

2. Which of the following is an assumption that supports drawing the conclusion above from the reasons given for that conclusion?

(A) Art critics generally have a different appraisal of a work of art than does the general public.

(B) The meaning of any work of art is defined entirely by the ideas of the artist who created it.

(C) Beauty is a defining quality of art.

(D) All art movements of the latter half of the twentieth century are responses to the movements of the first half of the century.

(E) It is not possible for any work to be simultaneously beautiful and intellectually engaging.

Click here for the answer and video explanation!

Most people can gain vitamin C from fruits such as oranges and cantaloupes. People with Laestrygonian Disease have weakened digestive systems that cannot digest fruit or vitamin supplements. The easiest foods for these people to digest are grains such as rice and barley. Regular intake of vitamin C would be extremely beneficial to those who suffer from Laestrygonian Disease, so scientists have figured out a way to create “fortified rice” by infusing rice with high doses of vitamin C. This fortified rice will provide great benefit to those with Laestrygonian Disease.

3. Which one of the following is an assumption on which the conclusion depends?

(A) Eventually, this fortified rice will be the optimal way for most people to have a regular intake of vitamin C.

(B) The problems that folks with Laestrygonian Disease have digesting fruit are different from their problems digesting vitamin supplements.

(C) People with Laestrygonian Disease will not be unable to assimilate the form of vitamin C that is present in the fortified rice.

(D) Only people whose diets consist largely of grains would be able to derive benefit from the vitamin C in the fortified rice.

(E) Vitamin C is the only nutrient which can be infused into rice in such high quantities without compromising the nutritional integrity of the vitamin.

Click here for the answer and text explanation
If folks with Laestrygonian Disease cannot assimilate the Vitamin C in the rice, then it won’t help them, and eating the fortified rice will not provide them any particular benefit. If we negate this option, it shatters the argument. This is a true assumption.

(A) This may be true, although I am skeptical that any human-made improved food would be better than the fruits designed by Nature! Regardless, whether this is true or not does not have any bearing on how helpful the fortified rice will be for the folks with Laestrygonian Disease. This option is incorrect.

(B) This is intriguing. Let’s negate this. Suppose it were the exact same problem, say, the exact same missing enzyme, that made it impossible to digest both fruit and vitamin supplements. Then what? Would that mean they also couldn’t digest the fortified rice, or get the vitamin C they need from it? We cannot say. It’s conceivable that the argument could still work, so negating this does not destroy the argument. This is not an assumption.

(D) Let’s negate this. Suppose the fortified rice benefits everyone—even the no-carbs fanatic who hasn’t touched carbs in a decade: even when this person breaks his carb-fast and has the fortified rice, he has benefit from it. What then? Whether these other people benefit or not from the fortified rice has no bearing on whether it helps the folks with Laestrygonian Disease. This choice is incorrect.

(E) Let’s negate this. Suppose we can infused dozens of other vitamins and minerals into the rice, all with high nutritional yield. That would only be good for the folks with Laestrygonian Disease—the more vitamins, the better! It certainly would not impact whether these folks derived any benefit from the vitamin C in the rice. This choice is incorrect.

The answer is (C).
Final Thoughts
Assumption questions will require you to read closely, but with practice you can identify the missing link. For more GMAT Critical Reasoning tips, check out our introduction to the CR section.

The post How to Tackle Critical Reading Assumption Questions appeared first on Magoosh Blog — GMAT® Exam.
This Blog post was imported into the forum automatically. We hope you found it helpful. Please use the Kudos button if you did, or please PM/DM me if you found it disruptive and I will take care of it. -BB
Magoosh GMAT Instructor
Joined: 28 Dec 2011
Posts: 4452
Own Kudos [?]: 28605 [0]
Given Kudos: 130
The Best GMAT Prep Courses: 2021 Edition with Comparison Charts [#permalink]
Expert Reply
FROM Magoosh Blog: The Best GMAT Prep Courses: 2021 Edition with Comparison Charts


Choosing the best GMAT prep course, like choosing a business school, is all about what works best for you as an individual. There is no objectively ‘best’ course that is right for everyone, but there are eight GMAT courses that should be on your radar in deciding which option suits your needs. One way to choose a course is to attend a free session or engage with some free material to see what suits you best.

To make your decision, you’ll have to consider your learning style, and which course elements—from price to score guarantees—matter most to you. Don’t worry. We’ve done the hard work for you and have broken down the pros and cons of what we consider the eight best GMAT online prep courses.

Magoosh

$249 (get 10% off here!)$110 per hour50+ score improvement guarantee1,300+ practice questions and two practice tests. Subscribers get a 40% discount on GMAT
Official Practice Exams

One-year access

Princeton Review$799$167 an hour 620+ starting score required to be eligible for the guarantee of a 700+ score3,000+ practice problems and 10 practice tests

Four-month access

Target Test Prep$99 per month to $399 for 6 months access$300 per hourScore improvement guarantee only applies to your quantitative score3,000+ practice questionsOne-month to four-month access

Manhattan Prep$549$225 per hourN/A1,100+ questions

and six practice tests

Six-month access

Kaplan$599 - $799$2,499 for 10 hoursScore improvement guarantee5,000+ questions and nine practice testsSix-month access

PrepScholar$2594 hours of tutoring from $799 60+ score guarantee Access to 1,000+ questions and

4 practice tests

One-month to four-month access

e-GMAT$349N/AN/A4,500+ questions and five practice testsSix-month access

Veritas Prep$699$2,650 for 10 hours 50+ score improvement guarantee5,000+ questions and 12 practice tests

One-year access









best GMAT books lists.

Pros:

  • Materials are engaging and tutors have a reputation for dynamic and entertaining lessons.
  • You get access to a discussion forum to delve into GMAT topics.
Cons:

  • There is no score guarantee.
  • Practice tests are known for being slightly more difficult than the GMAT itself. This could be a pro or a con depending on what motivates you.

 





computer adaptive practice tests[/list]
Review

PrepScholar’s GMAT prep course is designed around personalization. Students take a diagnostic test upfront, and their proprietary algorithm works to create a custom study plan. You can also see which skills you have mastered and which you still need to work on on your student dashboard.

Pros:

  • The personalized curriculum is perfect for those studying under time pressure.
  • There is a 60+ score guarantee—the most impressive of our ranked GMAT test prep courses. Note that you can only use an official test as a baseline.
Cons:

  • PrepScholar does not have as many practice questions as its competitors. To be fair though, few students get through all the practice questions on offer, in addition to those in the Official guide.
  • Your access period for PrepScholar is well below those of competitors—you get only four months of access for their flagship Completely Customised Online GMAT Prep course.





GMAT idioms flashcards app from Magoosh gives you all the most tested idioms specific to the GMAT, in an easy-to-learn flashcard format.

Speaking of easy-to-learn flashcard formats, the Magoosh Math flashcards give you an overview of all the GMAT math rules. While math is very much about practice, these flashcards make the general rules that you’ll need for Data Sufficiency more explicit and easier to remember.

If you are looking for a mental math workout before the test, look no further than the Math Tricks app and Calculator: The game app which allow you to practice the mental math tricks that can speed up your problem-solving in the GMAT. While these are not GMAT apps per se, they will help you to solve mental math problems more quickly and creatively.

The Wiley Efficient Learning mobile app also needs a mention as a must-have GMAT app. It is essentially a more portable version of the Official Guide. It will give you access to the same Official Guide Questions in the book and online question bank, just on the go.

 

What materials should I use in my GMAT prep?

Show Answer
You’ll want to get a copy of the GMAC Official guide so you can practice with real GMAT questions. But you’ll also need some guidance on how to answer the questions. For insights, tools, and strategies, you should get hold of some kind of GMAT prep books or courses. Don’t underestimate the usefulness of a GMAT study schedule. Learn how to build your own GMAT study schedule to keep yourself motivated and accountable.

Remember the best GMAT prep materials are the ones that work best for you. Part of the emphasis on practice is to get a sense of what works for you. Everyone is slightly different in their approach so experiment while you are practicing to adapt generic strategies to your benefit.

Whichever method of study you choose, try not to get too bogged down in the theory. Don’t learn the theory in isolation without considering how the GMAT tests different concepts. The GMAT is all about practice—and gaining as much insight into the test and the question types as you can from each question you practice.

In your first phase of studying you might find it useful not to focus too heavily on timing. Rather, learn the concepts at your own pace. Once you have a good idea of how to answer questions you can introduce timing in the second phase. Lastly, you should have practiced a number of mock tests before you take the GMAT to get a feel of the test, build up stamina and see what checks and balances you can introduce to improve your score.

 

These are our top picks for the best GMAT online prep courses! Like we mentioned earlier, the best GMAT prep course may vary greatly depending on your study style, strengths and weaknesses, and even your reason for taking the GMAT. Nevertheless, we hope this list can help you decide which prep course or study method works best for you. Good luck!

The post The Best GMAT Prep Courses: 2021 Edition with Comparison Charts appeared first on Magoosh Blog — GMAT® Exam.
This Blog post was imported into the forum automatically. We hope you found it helpful. Please use the Kudos button if you did, or please PM/DM me if you found it disruptive and I will take care of it. -BB
Magoosh GMAT Instructor
Joined: 28 Dec 2011
Posts: 4452
Own Kudos [?]: 28605 [0]
Given Kudos: 130
How to Understand Your GMAT Percentiles [#permalink]
Expert Reply
FROM Magoosh Blog: How to Understand Your GMAT Percentiles


How can you turn test scores and section scores into GMAT score percentiles? And what about GMAT Quant score percentiles and Verbal score percentiles? Don’t worry—we’ve got all the info you need to know to understand your score. First off, your total GMAT score ranges from 200-800, and your section scores have their own score scales as well. But focusing on your GMAT percentiles can give you extra insight into your strengths and weaknesses on the test.

The GMAT is all about competitive advantage. Your score is literally determined by how well you perform in relation to hundreds of thousands of test-takers over the past three years. In a nutshell, a ranking in the 75th percentile means that 25% of test-takers performed as well or better than you, and 75% did not.

Knowing the percentiles lets you know if you’ve merely achieved an average GMAT score, or if you’ve shot through the roof with your scores. GMAC produces this information on GMAT percentile charts. For your convenience, we’ve broken these scores down below.

This post has been updated to include GMAC’s most recently published GMAT score percentiles, which represent a sample of nearly 700,000 students who took the GMAT from 2017-2019.

GMAT Score Percentiles Chart[/*]
[*]GMAT Quant Percentiles[/*]
[*]GMAT Verbal Percentiles[/*]
[*]GMAT AWA Percentiles[/*]
[*]GMAT IR Percentiles[/*]
[*]How does GMAT calculate percentiles?
[*]GMAT Percentiles for Business Schools[*]Takeaways[/list]
information on GMAT score calculations!)

Click here for the GMAT Score Percentiles Chart

PercentileScorePercentile (cont'd.)Score (cont'd.)

99%760-80030%520

98%75027%510

97%74025%500

96%73023%490

94%72021%480

91%71019%470

88%70017%460

85%69015%450

82%68014%440

80%67013%430

77%66012%420

73%65010%410

67%6409%400

65%6308%390

62%6208%380

58%6107%370

54%6006%360

51%5905%350

47%5805%340

44%5704%320-330

41%5603%300-310

38%5502%260-290

35%5401%220-250

31%5300%200-210

 















MBA.com are the best way to get an accurate idea of how you are performing before test day.

The one-time thinking about the scoring algorithm, percentiles, and the difficulty level of questions is super unhelpful is during the test itself. It’s a waste of mental energy and effort, in part due to the experimental questions in the test, that doesn’t count towards your total score.



as they did in 2017-2018), that means around:

  • 2,500 scored between 760-800 (99th percentile)
  • 2,500 scored between 750-760 (98th percentile)
  • 25,000 scored between 700-750 (88th-98th percentile)
Starting to see a trend? Yep! The GMAT scores fall along a bell curve. Very few people get very high or very low scores, and most people fall somewhere in the middle. In fact, the GMAC tells us that around two-thirds of test-takers score between 400 and 600 on the exam.



retake the GMAT (and, unfortunately, pay the GMAT exam fee once more).

Bear in mind that some schools will have minimum sectional scores for the Quant section as well. Others may insist on a ‘balanced score’ between the two sections.

To understand the GMAT score you’ll need to obtain for a top school, the best thing to do is look at average GMAT scores for top programs. As you can see from running down this list, they all tend to be above 700–sometimes well above 700, as in the case of Columbia (an average of 732!).

Put this in terms of percentiles using the chart above. A 700 on the GMAT is in the 88th percentile, while a 732 would be around the 96th percentile. From this information, we can draw the conclusion that top schools need top scores.

Rankings don’t correspond precisely to score percentiles—but in this case, it’s pretty fair to say that if you’re applying to a top-10 school, a score in the top 10% of GMAT test-takers (i.e. placing you in the 90th percentile above) is definitely a helpful tool.



How to Understand Your GMAT Percentiles appeared first on Magoosh Blog — GMAT® Exam.
This Blog post was imported into the forum automatically. We hope you found it helpful. Please use the Kudos button if you did, or please PM/DM me if you found it disruptive and I will take care of it. -BB
Magoosh GMAT Instructor
Joined: 28 Dec 2011
Posts: 4452
Own Kudos [?]: 28605 [0]
Given Kudos: 130
GMAT Test Dates | 2021, 2022, 2023 and Beyond! [#permalink]
Expert Reply
FROM Magoosh Blog: GMAT Test Dates | 2021, 2022, 2023 and Beyond!
[img]https://magoosh.com/gmat/files/2017/02/GMAT-Test-Dates.jpg[/img]

With the COVID-19 still a concern in 2021, [url=https://www.mba.com/service/testing/appointment-search]in-person[/url] GMAT test dates are limited, but [url=https://www.mba.com/exams/gmat-online/gmat-online-experience/about-the-gmat-online]at-home[/url] options are available—and will be permanently available going forward. In this post, we’ve covered some more vital information about the GMAT test dates for 2021—read on to find out more!

[url=https://magoosh.com/gmat#dates]GMAT Test Dates (2021, 2022, 2023)[/url][/*]
[*][url=https://magoosh.com/gmat#testcenter]The 3 Steps to Scheduling Your GMAT Test Date[/url][/*]
[*][url=https://magoosh.com/gmat#appdates]MBA application dates[/url]
[list]
[*][url=https://magoosh.com/gmat#round1]Round 1[/url][/*]
[*][url=https://magoosh.com/gmat#round2]Round 2[/url][/*]
[*][url=https://magoosh.com/gmat#round3]Round 3[/url][/*]
[/list]
[*][url=https://magoosh.com/gmat#register]When do I need to register for the GMAT?[/url][/*]
[*][url=https://magoosh.com/gmat#plan]When should I plan on taking the GMAT?[/url]
[/list]
[url=https://magoosh.com/gmat/gmat-online/]online, at-home GMAT[/url] testing options. Meanwhile, the GMAC (test-maker) began to offer the remote GMAT in April 2020; it’s offered every day, with multiple time slots.

However, popular days and times can still fill up—the remote proctor aspect of the testing means that there are still limits to how many test-takers can sign up for the exam at a given time.

With all of these GMAT test dates to choose from, should you just throw a dart at a calendar? No! You’ll ned to be strategic about picking your test date, because you’ll need your score for whatever MBA application round you choose. We’ll talk more about this later i the post!

[url=https://magoosh.com/gmat#contents][img]https://magoosh.com/gmat/files/2017/07/back-to-top-button-1-150x40.png[/img][/url]

[url=https://accounts.gmac.com/Account/Register]GMAC’s “Create Your Account”[/url] page to get started. Once that’s done, you’ll have the ability to browse and see your options for GMAT dates (and locations, if applicable).

Step 2a: Find a test center with available GMAT dates
Once you’ve registered with GMAC and have the ability to look at GMAT exam dates, the next step is to find a GMAT test center. For in-person dates, look under “Future GMAT Test Center Exams” and click “Register Now” or “Register as a Test Taker with Disabilities.” Each of these buttons will take you to options for your location.

Remember, the farther in advance you book your GMAT, the more options you’ll have for both GMAT locations and GMAT dates.

Step 2b : Pick a date for the online GMAT
Because the online GMAT is available every day, the key to picking a GMAT test date online is finding the day and time that work best for you.

To find test dates for a remote GMAT, select “Register for the GMAT Online Exam” under “Future GMAT Online Exams” on your MBA.com “My Account” page. Click on dates to see available time options.

Step 3: Schedule Your Exam
Once you know which test format you want to take and are registered to take the GMAT, you’re ready to actually select your GMAT test date. Follow the on-screen instructions within your GMAT account.

Of course, there’s a lot more to registering for the GMAT and choosing your test date than just accessing and navigating the official GMAT website. Timing is a crucial consideration too. Below, we’ll look at when to register, and how to select the GMAT test date that’s right for you.

[url=https://magoosh.com/gmat#contents][img]https://magoosh.com/gmat/files/2017/07/back-to-top-button-1-150x40.png[/img][/url]

[url=https://magoosh.com/blog/live-updates-effects-of-coronavirus-on-standardized-testing/]CANCELLED in many locations in 2020[/url], but [url=https://magoosh.com/gmat/online-interim-gmat-exam/]online, home-based GMAT testing[/url] is available now.StudyRetake GMATEssays, etc...Round 1 due

[url=https://magoosh.com/gmat#contents][img]https://magoosh.com/gmat/files/2017/07/back-to-top-button-1-150x40.png[/img][/url]

[url=https://magoosh.com/gmat/gmat-101/]GMAT examination[/url].

[url=https://magoosh.com/gmat#contents][img]https://magoosh.com/gmat/files/2017/07/back-to-top-button-1-150x40.png[/img][/url]

[url=https://www.mba.com/us/the-gmat-exam/gmat-exam-scores/your-score-report/send-your-scores.aspx]GMAC says[/url] it can take up to 20 days for your test to be reported to schools. That could actually mean 21 days, depending on whether your test day scores get transmitted from the test center to GMAT central the same day you take the test or the next day. There’s also a small chance that for some reason there could be unexpected delays in the GMAC sending our scores. So, beyond GMAT exam dates, you should consider the dates of arrival for your GMAT exam report.

To be safe, you’ll want to book your GMAT date at least 21 days prior to your application deadline, so that there is ample time for your scores to be processed and sent to your school. Giving yourself a few extra days beyond the 21 can’t hurt either. So really, there’s an extra month in there. That’s why I recommend taking your GMAT 7-8 months before your deadline instead of just 6-7.

Adjust the recommended GMAT application timeline to your personal needs
Of course, your mileage may vary, in terms of how much time you really need and when you should register for the GMAT. If you have more than 20 hours a week to study or have higher-than-average GMAT skills, you may need less time to prepare for your exam or a possible retake. Conversely, if you have fewer hours of spare time per week or lower baseline skills for the exam, you may need to reserve more prep time before you actually take the test.

Also, while it isn’t as common, some students get all of the soul searching, university visits, reference letter gathering and so on out of the way before they tackle the GMAT. If this sounds like you, then your GMAT prep time and GMAT exam date will take a different position in your overall timetable for admissions.

[url=https://magoosh.com/gmat#contents][img]https://magoosh.com/gmat/files/2017/07/back-to-top-button-1-150x40.png[/img][/url]

The post [url=https://magoosh.com/gmat/gmat-test-dates/]GMAT Test Dates | 2021, 2022, 2023 and Beyond![/url] appeared first on [url=https://magoosh.com/gmat]Magoosh Blog — GMAT® Exam[/url].
This Blog post was imported into the forum automatically. We hope you found it helpful. Please use the Kudos button if you did, or please PM/DM me if you found it disruptive and I will take care of it. -BB
GMAT Club Bot
GMAT Test Dates | 2021, 2022, 2023 and Beyond! [#permalink]
   1  ...  3   4   5   6   7   8   9   10   11   12   13   14   

Powered by phpBB © phpBB Group | Emoji artwork provided by EmojiOne